Sunteți pe pagina 1din 88

NBME FORM 2

Block 1

1.) A 50-year-old man with a 20-year history of type


2 diabetes mellitus has had sensory neuropathy for 2 weeks.
Pulses are decreased at the ankle. There is no peripheral edema. His serum
glucose levels have been between 150 mg/dL and 200 mg/dL over the past 6
months. Which of the following is the most effective measure to prevent
serious foot infections?
A) Use of support hose
B) Use of well-fitted shoes
C) Prophylactic antibiotics
D) Daily aspirin and dipyridamole
E) Decrease in serum glucose level
2.) A 63-year-old woman comes to the emergency
department because of palpitations for 1 hour. She was diagnosed with
hyperthyroidism 2 months ago but has not received treatment. Three
hours before arrival, she drank two bottles of beer with dinner. Her blood
pressure is 90/60 mm Hg, and pulse is 85/min and irregularly irregular.
Examination shows a variation in intensity of S1. There is an absence of
waves in the venous pulse. An ECG is shown. Which of the following is
the most likely cardiac rhythm disturbance?
A) Atrial fibrillation
B) Atrial flutter
C) Multifocal atrial tachycardia
D) Paroxysmal atrial tachycardia
E) Second-degree atrioventricular block, Wenckebach type
F) Sick sinus syndrome
G) Sinoatrial exit block, type II
H) Torsades de pointes
I) Wandering pacemaker
J) Wolff-Parkinson-White syndrome

3.) A 5-year-old girl is brought to the physician 30 minutes after


being bitten on the forearm by her cat. Examination shows a small
puncture wound covered with dried blood. She is at increased risk of
infection for which of the following reasons?
A) Arm wounds are more susceptible to infection
B) Salivary enzymes in cats augment infection
C) It is difficult to cleanse pathogens from this wound
D) The Staphylococcus species involved is particularly virulent
E) Young children are less able to resist infection than adults
4.) A 4-year-old girl is brought to the physician because of pallor
and jaundice for 2 days. She had previously been well, although she was
treated for jaundice with phototherapy for 2 weeks while a newborn.
Her mother and two additional maternal relatives underwent splenectomy
during childhood for unknown reasons. Examination of the patient shows
jaundice. The spleen tip is palpated 4 cm below the left costal margin.
Which of the following blood smear findings is most likely to explain this
family's condition ?
A) Elliptocytes
B) Howell-Jolly bodies
C) Schistocytes
D) Sickle cells
E) Spherocytes
5.) A sexually active 20-year-old woman has had fever, chills, malaise, and pain
of the
vulva for 2 days. Examination shows a vulvar pustule that has ulcerated and
formed multiple satellite lesions. Nodes are palpated in the inguinal and
femoral areas. A smear of fluid from the lesions establishes the diagnosis.
Which of the following is the most likely causal organism?
A) Chlamydia trachomatis
B) Haemophilus ducreyi
C) Neisseria gonorrhoeae
D) Streptococcus pyogenes (group A)
E) Treponema pallidum
6.) A 5-year-old boy is brought to the physician 1 hour after
urinating bright red blood. He has been taking ibuprofen since injuring his
right flank while wrestling with friends yesterday; he also has been
taking penicillin for 3 days for streptococcal pharyngitis. His
temperature is 36.7 C (98 F), blood pressure is 90/48mm Hg, pulse is 108/min,

and respirations are 18/min. Examination shows purple ecchymoses over


the shins and right flank; there is tenderness of the right
costovertebral area. The abdomen is nontender. Genital examination shows no
abnormalities. There is no edema. Urinalysis shows gross blood;
microscopic examination shows 5 10 leukocytes/hpf and erythrocytes that
are too numerous to count. Which of the following is the most likely
explanation for this patient's hematuria?
A) Acute pyelonephritis
B) Ibuprofen-induced acute papillary necrosis
C) Post-streptococcal glomerulonephritis
D) Rhabdomyolysis
E) Traumatic injury to the kidney
7.) A 16-year-old girl comes to the physician for her first prenatal
visit at 12 weeks' gestation. She has not had any immunizations since
the age of 5 years. She has received the following immunizations at the
recommended ages:
5
3
1
4

Diphtheria-tetanus-pertussis
Hepatitis B
Measles-mumps-rubella
Oral poliovirus

Examination shows no abnormalities. Which of the following is the most


appropriate immunization to administer at this visit?
A) Diphtheria-tetanus-pertussis
B) Diphtheria (child)-tetanus toxoid
C) Diphtheria (adult)-tetanus toxoid
D) Haemophilus influenzae type b
E) Hepatitis B
F) Inactivated poliovirus
8.) A 12-year-old boy is brought to the physician because of
temperatures to 40.2 C (104.4 F), lethargy, and a stiff neck for 2 days. A
diagnosis of pneumococcal meningitis is made, and the child is admitted to
the hospital for antibiotic therapy. Serum studies 24 hours after
admission show:
Na+ 117 mEq/L
Cl 89 mEq/L
HCO3 25 mEq/L
Which of the following is the most likely explanation for these findings?
A) Excessive cortisol release
B) Hyperaldosteronism

C) Inappropriate secretion of ADH (vasopressin)


D) Increased production of leukotriene C4
9. )An asymptomatic 57-year-old man with a 3-year history of type 2
diabetes mellitus comes for a routine follow-up visit. Examination shows no
abnormalities. Serum studies show:
Aspartate aminotransferase (AST, GOT) 76 U/L
Alanine aminotransferase (ALT, GPT) 86 U/L
Iron 260 g/dL Total iron-binding capacity 300 g/dL (N=250 450)
Ferritin 1200 ng/mL Antinuclear antibody negative
Serologic testing for hepatitis is negative. Which of the following is
the most appropriate next step in management?
A) Corticosteroid therapy
B) Interferon therapy
C) Penicillamine therapy
D) Chronic phlebotomy
E) No therapy indicated
10.) A 26-year-old man has had anxiety and insomnia since he lost his
job 1 month ago. He also has had palpitations, daily headaches, and
flatulence. He has no history of psychiatric disorders. His mother has
a history of bipolar disorder, mixed, and his father has alcoholism;
his younger sister had a history of drug abuse but has been abstinent
from drugs for 3 years. Which of the following is the most likely
diagnosis?
A) Adjustment disorder
B) Bipolar disorder, depressed
C) Generalized anxiety disorder
D) Major depressive disorder
E) Post-traumatic stress disorder

11.) A previously healthy 32-year-old man comes to the emergency


department because of a 12-hour history of fatigue and shortness of breath
at rest. He takes no medications. His temperature is 7 C (98.6 F), blood pressur
e is
96/58 mm Hg, pulse is 110/min, and respirations are 22/min. Examination shows
scleral icterus and conjunctival pallor. There is no hepatosplenomegaly. Laborat
ory
Studies show:
Hemoglobin 6.2 g/dL
Serum Leukocyte count 8800/mm3
Bilirubin, total 8.5 mg/dL
Reticulocyte count 16%
Direct 1.5 mg/dL

Platelet count 245,000/mm3


Lactate dehydrogenase 1260 U/L
A) Acute toxic hepatitis
B) Alcoholic hepatitis
C) Cholangiocarcinoma
D) Chronic idiopathic cirrhosis
E) Common bile duct calculus
F) Congestive hepatitis
G) Gallstone pancreatitis
H) Gilbert's syndrome
I) Infectious hepatitis
J) Intravascular hemolysis
K) Primary biliary cirrhosis
12.) A 24-year-old third-year medical student comes to the student health clinic
for
evaluation of jaundice which he noticed this morning. He has had no abdominal
pain, itching, or weight change. He takes no medications. He describes recent
anxiety over anticipation of the upcoming Step 2 examination. His temperature is
37 C (98.6 F), blood pressure is 120/80 mm Hg, pulse is 76/min, and respirations
are 12/min. Examination shows no abnormalities except for scleral icterus.
Laboratory studies show:
Hemoglobin 15 g/dL Serum
Leukocyte count 7000/mm3
Alkaline phosphatase 90U/L
Platelet count 240,000/mm3
Aspartate aminotransferase Serum (AST, GOT) 23 U/L
Bilirubin, total 3.5 mg/dL
Alanine aminotransferase Direct 0.7 mg/dL (ALT, GPT) 27 U/L
A) Acute toxic hepatitis
B) Alcoholic hepatitis
C) Cholangiocarcinoma
D) Chronic idiopathic cirrhosis
E) Common bile duct calculus
F) Congestive hepatitis
G) Gallstone pancreatitis

H) Gilbert's syndrome
I) Infectious hepatitis
J) Intravascular hemolysis
K) Primary biliary cirrhosis
13.) A 37-year-old woman with a 2-year history of hypertension comes for a follo
w-up
examination; her hypertension has worsened despite treatment with a low-sodium
diet and a -adrenergic blocking agent. She has a history of rheumatic fever and
Graves' disease treated with 131I. Her blood pressure is 160/106 mm Hg, and
pulse is 80/min. Serum studies show:
Na+ 135 mEq/L
Cl 100 mEq/L
K+ 3.4 mEq/L
HCO3 26 mEq/L
Urea nitrogen (BUN) 12 mg/dL
Creatinine 0.8 mg/dL
Urinalysis is within normal limits. Renal ultrasonography shows a left kidney wi
th a
markedly irregular contour; it is 2.8 cm smaller than the right.
A) Adrenal cortex
B) Adrenal medulla
C) Aorta
D) Renal arteries
E) Renal glomeruli
F) Thyroid gland
14.) A 27-year-old woman comes to the physician because of muscle weakness and
cramps for 2 weeks. She has been taking a -adrenergic blocking agent for
hypertension for 2 years. She had chronic lymphocytic thyroiditis (Hashimoto's
disease) 1 year ago. Her blood pressure is 160/108 mm Hg, and pulse is 60/min.
Serum studies show:
Na+ 140 mEq/L
Cl 110 mEq/L
K+ 2.2 mEq/L
HCO3 30 mEq/L
Urea nitrogen (BUN) 20 mg/dL
Creatinine 1 mg/dL
Magnetic resonance angiography of the abdomen shows normal findings.
A) Adrenal cortex
B) Adrenal medulla
C) Aorta

D) Renal arteries
E) Renal glomeruli
F) Thyroid gland
15.) A 24-year-old man is hospitalized for treatment of a posterior dislocation
of the
right knee sustained in a motorcycle collision. Six hours after closed reduction
,
previously present distal pulses in his foot are absent, but the foot has remain
ed
warm. Which of the following is the most appropriate next step in management?
A) Elevation of the limb and observation for 24hours
B) Nitroprusside therapy
C) Sympathetic block
D) Femoral arteriography
E) Embolectomy with a Fogarty catheter through aproximal arteriotomy
D) Inflammation of the origin of the plantar fascia

16.) A healthy 4-year-old girl is brought for a well-child examination. A grade


2/6
systolic ejection murmur is heard along the upper left sternal border. S2 is wid
ely
split and does not vary with respiration. A soft mid-diastolic murmur is heard a
long
the lower left sternal border. Examination shows no other abnormalities. Which o
f
the following is the most likely diagnosis?
A) Aortic stenosis
B) Atrial septal defect
C) Coarctation of the aorta
D) Mitral valve prolapse
E) Patent ductus arteriosus
F) Pulmonary stenosis
G) Tetralogy of Fallot
H) Transposition of the great arteries
I) Ventricular septal defect

J) Normal heart

18.) A previously healthy 57-year-old woman comes to the physician because of th


ree
episodes of blurred vision in the right eye over the past 3 weeks; each episode
lasts approximately 5minutes. Retinal examination shows a small refractile body
at
The bifurcation of a retinal artery. The remainder of the examination shows no
abnormalities. Which of the following is the most appropriate next step in
diagnosis?
A) Cerebral angiography
B) Echocardiography
C) Electroencephalography
D) Duplex scan of the carotid arteries
E) MRI of the brain
19.) A 67-year-old woman comes to the physician for her first influenza virus
vaccination. She has a history of untreated hypertension. Her blood pressure is
160/100 mm Hg, and pulse is 100/min. Shortly after administration of the
influenza virus vaccine, she develops shortness of breath, hives, and angioedema
.
Which of the following is most likely to have prevented this reaction?
A) Inquiry about an egg allergy
B) Heterophile agglutination test
C) Skin test with histamine reagent
D) -Adrenergic blocking agent therapy
E) Amantadine therapy
F) Insulin therapy
G) Rimantadine therapy

20.) A 7-year-old girl is brought to the physician because of a 2-day history of


fever,
headache, sore throat, and swollen glands. She does not have a runny nose,
congestion, or cough. She has no allergies to medications. Her temperature is
38.6C (101.4 F), blood pressure is 100/60 mm Hg, pulse is 120/min, and
respirations are 16/min. Examination shows a swollen, erythematous oropharynx
With tonsillar exudates. The anterior cervical lymph nodes are enlarged and
tender. No other abnormalities are noted. Which of the following is the most lik
ely
causal organism?

A) Adenovirus
B) Corynebacterium diphtheriae
C) Group A streptococcus
D) Haemophilus influenzae
E) Mycoplasma pneumoniae
21.) A 70-year-old nursing home resident is admitted to the hospital because of
progressive obtundation over the past 2 days. He has tachycardia, tachypnea, and
hypotension. Bilateral basilar crackles and an S3 gallop are heard on auscultati
on.
Examination shows jugular venous distention and peripheral edema. Swan-Ganz
catheterization shows a cardiac index of 1.8 L/min/m2 (N=2.5 4.2), a mean
Pulmonary capillary wedge pressure of 23 mm Hg (N=1 10), and markedly
Increased systemic vascular resistance. Which of the following is the most likel
y
diagnosis?
A) Cardiogenic shock
B) Hypovolemic shock
C) Neurogenic shock
D) Septic shock

22.) A 42-year-old man with alcoholism is brought to the emergency department by


a
friend because of fever and progressive shortness of breath for 12 hours. His
friend reports that they were eating leftover chicken and drinking beer earlier
in
the day when the patient suddenly choked and vomited. Six hours later, he
developed sweating, chills, and shortness of breath. He has not had any
subsequent nausea, vomiting, hematemesis, or abdominal pain. He has smoked
Two packs of cigarettes daily for 22 years and drinks 12 to 18 beers daily. He i
s in
moderate respiratory distress. His temperature is 39.3 C (102.8 F), blood pressu
re
is 90/60 mm Hg, pulse is 120/min, and respirations are 24/min. Examination
shows no jugular venous distention. Breath sounds are decreased halfway up the
left lung with increased dullness. Cardiac examination shows a normal S1 and S2;
no murmurs are heard. There is no abdominal tenderness. Bowel sounds are
hypoactive. X-ray films of the chest show a left pleural effusion and air in the
mediastinum. Thoracentesis is performed.
Laboratory studies show:
Leukocyte count 18,000/mm3
Segmented neutrophils 85%
Bands 10%
Lymphocytes 5%
Serum Protein 6 g/dL
Lactate dehydrogenase 200 U/L

Pleural fluid Leukocyte count 8000/mm3


Segmented neutrophils 98%
Monocytes 2%
Protein 4.2 g/dL
Amylase 140 U/L
Lactate dehydrogenase 180 U/L
Gram's stain
WBC present
Organisms none
Which of the following is the most likely diagnosis?
A) Congestive heart failure
B) Esophageal rupture
C) Pancreatitis
D) Pericarditis
E) Tuberculosis
23.) A 2-year-old girl with tricuspid atresia has increasing respiratory distres
s for 2
days. She has been recovering uneventfully from an operation 10 days ago to join
systemic venous return with pulmonary arterial circulation. Over the past 4 days
,
she has been weaned off mechanical ventilation, started on oral feedings, and
is receiving chest physiotherapy for atelectasis. Her temperature is 37.4 C (99.
3 F),
blood pressure is 98/64 mm Hg, pulse is 120/min, and respirations are 46/min.
Examination shows nasal flaring, grunting, and intercostal retractions. An x-ray
film of the chest shows large bilateral pleural effusions. Thoracentesis yields
300
mL of whitish-yellow fluid. The supernatant remains uniformly opaque on
centrifugation. Which of the following is the most likely cause of the pleural
effusions?
A) Chylothorax
B) Congestive heart failure
C) Empyema
D) Pulmonary embolism
E) Superior vena cava obstruction
24.) A 23-year-old man is brought to the physician by his mother because he has
heard
a voice over the past month telling him to hurt himself. His mother says that he
r
son has no friends and is a lifelong loner; since graduating from high school, h
e
has been unable to hold a job. He admits to smoking marijuana occasionally and
drinking six beers weekly. Examination shows a poorly groomed man with poor

eye contact. He has a flat affect and limited facial expression. He says he has
no
intention of harming himself or others. Which of the following is the most
appropriate next step in management?
A) Schedule a follow-up visit in 4 weeks
B) Prescribe oral risperidone and schedule a follow-up visit in 2 weeks
C) Admit him to the partial hospital program and prescribe oral lithium carbonat
e
D) Admit him to the psychiatric unit for detoxification
E) Admit him to the psychiatric unit and prescribe oral imipramine

25.) A 23-year-old man is brought to the physician by his mother because he has
heard
a voice over the past month telling him to hurt himself. His mother says that he
r
son has no friends and is a lifelong loner; since graduating from high school, h
e
has been unable to hold a job. He admits to smoking marijuana occasionally and
Drinking six beers weekly. Examination shows a poorly groomed man with poor
Eye contact. He has a flat affect and limited facial expression. He says he
has no intention of harming himself or others. Which of the following is the mos
t
appropriate next step in management?
A) Schedule a follow-up visit in 4 weeks
B) Prescribe oral risperidone and schedule a follow-up visit in 2 weeks
C) Admit him to the partial hospital program and prescribe oral lithium carbonat
e
D) Admit him to the psychiatric unit for detoxification
E) Admit him to the psychiatric unit and prescribe oral imipramine

26.) An 18-month-old boy is brought for a well-child examination. He was born at


37
weeks' gestation and weighed 2800 g (6 lb 3oz). There were no prenatal or
perinatal complications. Developmental history indicates that he is able to run
and
walk up stairs while holding his mother's hand. He can hold a crayon but does no
t
Scribble spontaneously. He is at the 40th percentile for length and 50th percent
ile
for weight. Examination shows a crude pincer grasp. Which of the following is th
e
most appropriate assessment of fine and gross motor development?

Fine Motor
Development
A)
B)
C)
D)

Delayed
Normal
Delayed
Norma

Gross Motor
Development
normal
delayed
delayed
l normal

27.) Five months after beginning fluoxetine to treat obsessive-compulsive disord


er, a
19-year-old man states that he discontinued his medication 2 months ago because
he had begun to worry about taking his medication every day. His initial respons
e
to the medication was good. His symptoms have now returned, and his morning
ritual of cleaning and grooming consumes so much time that his job is in jeopard
y.
In addition to education about the nature of his disorder and its treatment, whi
ch
of the following is the most appropriate next step in management?
A) Tell the patient to schedule a return visit as needed
B) Offer to change the medication
C) Request that the patient's parents superviseadministration of medication
D) Monitor the patient's compliance by weekly blood tests
E) Begin a trial of cognitive-behavior therapy

28.) A 32-year-old woman comes to the physician because of a 3-month history of


increasing pain and stiffness in her wrists, hands, and ankles. During this peri
od,
she also has had progressive fatigue and morning stiffness lasting 2 hours. She
has
a 1-year history of rheumatoid arthritis treated with naproxen. Examination show
s
redness, swelling, and warmth over the wrist, hand, and ankle joints bilaterally
.
There are nontender subcutaneous nodules over the extensor surfaces of both
elbows. X-ray films of the hands show diffuse osteopenia and erosions over sever
al
of the distal metacarpal bones. Which of the following is the most appropriate
pharmacotherapy?
A) Add oral cyclophosphamide
B) Add oral gold
C) Add oral methotrexate
D) Add oral penicillamine
E ) Switch to oral ibuprofen

28.) A previously healthy 62-year-old man comes to the physician because of a 2month
history of progressive shortness of breath and a mild nonproductive cough. He
does not smoke. He worked in a foundry most of his adult life before retiring 2
years ago. Vital signs are within normal limits. Crackles are heard at both lung
Bases with no wheezes. Cardiac examination shows an accentuated P2. The
remainder of the examination shows no abnormalities. An x-ray film of the chest
shows prominent interstitial markings at the lung bases. Echocardiography shows
an ejection fraction of 55%. Pulmonary function testing is most likely to show
which of the following?
A) Decreased FEV1:FVC ratio
B) Decreased maximal inspiratory effort
C) Decreased total lung capacity
D) Increased forced vital capacity
E) Normal carbon monoxide diffusion capacity

29.) A 55-year-old man is admitted to the hospital because of progressive shortn


ess of
breath for 10 days. He has a history of chronic venous stasis and deep venous
thrombosis. He has been treated with warfarin since he had several pulmonary
emboli 2 years ago; he takes no other medication. He has smoked two packs of
Cigarettes daily for 30 years. He weighs 109 kg (240 lb) and is 165 cm (65 in) t
all.
Examination shows jugular venous distention. Echocardiography is most likely to
show which of the following?
A) Left ventricular dilation
B) Mitral stenosis
C) Pericardial thickening and tamponade
D) Right ventricular hypertrophy and dilation
E) Tricuspid stenosis
30.) A 32-year-old woman comes to the physician because of weakness of the lower
extremities for 2 days. Three years ago, she had pain and partial loss of vision
of
the right eye; the vision returned to normal after 6 weeks. There is mild pallor
of
the right optic disc. She has impaired tandem gait. Babinski's sign is present
bilaterally. There is mild spasticity of the lower extremities and mild weakness
of
the iliopsoas and hamstring muscles. Serum creatine kinase activity is 50 U/L.
Which of the following is the most likely diagnosis?

A) Amyotrophic lateral sclerosis


B) Becker's muscular dystrophy
C) Diffuse sensorimotor peripheral neuropathy
D) Duchenne's muscular dystrophy
E) Hyperkalemic periodic paralysis
F) Hypokalemic periodic paralysis
G) Multiple sclerosis
H) Myasthenia gravis
I) Myasthenic (Lambert-Eaton) syndrome
J) Myotonic muscular dystrophy
K) Polymyositis
L) Pontine astrocytoma
M) Pontine infarction
N) Spinal cord tumor
O) Transverse myelitis

31.) A healthy 42-year-old man comes to the physician for a life insurance evalu
ation.
He smoked one-half pack of cigarettes daily for 20 years but quit 10 years ago.
His
father died of a myocardial infarction at the age of 65 years. The patient weigh
s 93
kg (205lb) and is 178 cm (70 in) tall. His blood pressure is 160/110 mm Hg,
pulse is 96/min, and respirations are 16/min. Physical examination, ECG, and an
xray film of the chest show no abnormalities. Laboratory studies are within norma
l
limits except for a serum cholesterol level of 206 mg/dL. Which of the following
is
the greatest risk factor for cerebral infarction in this patient?
A) Genetic profile
B) History of smoking
C) Hypercholesterolemia
D) Hypertension
E) Obesity

32.) A previously healthy 56-year-old woman comes to the physician because of


jaundice and dark urine for 3 weeks. She has a 1-year history of generalized
pruritus. She takes no medications. Examination shows jaundice and several
ecchymoses over the forearms and thighs. The liver and spleen are enlarged and
nontender. Laboratory studies show:
Prothrombin time 18 sec
Serum Protein Total 8.5 g/dL
Albumin 3.8 g/dL
Bilirubin Total 5 mg/dL
Direct 2 mg/dL
Alkaline phosphatase 150 U/L
Alanine aminotransferase (ALT, GPT) 45 U/L
Antimitochondrial antibody assay is strongly positive.
A CT scan of the abdomen shows hepatosplenomegaly.
Endoscopic retrograde cholangiopancreatography shows no abnormalities.
Because of her condition, this patient is at greatest risk for which of the foll
owing
deficiencies?
A) Niacin
B) Vitamin A
C) Vitamin B2 (riboflavin)
D) Vitamin B12 (cyanocobalamin)
E) Vitamin C
33.) An asymptomatic 52-year-old man comes for a
follow-up
examination 1 month after he passed renal calculi. He
has a history of renal
calculi 2 years ago. Serum uric acid and calcium
levels and urinary
oxalate excretion are within normal limits. Urinary
calcium excretion is
increased. In order to avoid recurrence of renal
calculi, which of the
following is the most appropriate pharmacotherapy for
this patient?
A) Bicarbonate
B) Calcium lactate
C) Methenamine mandelate
D) Probenecid
E) Thiazide diuretic
34.
A 57-year-old woman is brought to the emergency

department because of
abdominal pain for 12 hours. Over the past 3 hours,
the pain has become
severe and generalized. Over the past month, she has
had mild upper
abdominal discomfort that is relieved by eating. She
has a history of
recurrent migraines treated with sumatriptan as
needed. Her temperature
is 38.2 C (100.8 F), blood pressure is 170/95 mm Hg,
and pulse is
110/min. Abdominal examination shows mild distention;
there is marked
rigidity with diffuse tenderness. Bowel sounds are
absent. Rectal
examination shows no abnormalities; test of the stool
for occult blood is
negative. Laboratory studies show:
Hematocrit 36%
Leukocyte count 16,500/mm3
Serum
Na+ 145 mEq/L
Cl 106 mEq/L
K+ 3.8 mEq/L
HCO3 19 mEq/L
Urea nitrogen (BUN) 32 mg/dL
Which of the following is the most appropriate next
step in diagnosis?
A) X-ray films of the abdomen while supine and
standing
B) Abdominal ultrasonography
C) Upper gastrointestinal series with contrast
D) HIDA scan
E) Fiberoptic endoscopy of the upper
gastrointestinal tract

35. An 18-year-old man comes to the physician


because of itchy
lesions on his penis for 2 weeks and itching around
his wrists and ankles
for 1 week. He is sexually active and does not
consistently use condoms.
Examination shows scattered, crusted lesions on the
penis and no
significant inguinal lymphadenopathy. There are
excoriated papules on the
wrists and ankles, a few papules between the fingers,
and excoriations
along the belt line. Which of the following is the
most likely

diagnosis?
A) Disseminated primary herpes simplex
B) Eczema
C) Scabies
D) Secondary syphilis
E) Vasculitis
36. An asymptomatic 62-year-old man comes for a
follow-up visit.
One month ago, he had acute cystitis treated with
ciprofloxacin. At his
initial visit, a urine culture grew Klebsiella
pneumoniae. He has a
2-year history of chronic prostatitis and has had four
episodes of
cystitis over the past year. His temperature is 36.9
C (98.4 F).
Examination, including prostate examination, shows no
abnormalities. Which of
the following is the most appropriate measure to
prevent recurrent
episodes of cystitis in this patient?
A) Trimethoprim-sulfamethoxazole prophylaxis
B) Insertion of a Foley catheter
C) Total prostatectomy
D) Transurethral prostatectomy
E) No treatment is available

37. A 32-year-old woman comes for a routine health


maintenance
examination. Examination shows a bloody discharge
from the nipple of her
left breast. There is no palpable mass. On
questioning, she says that
she never noted the discharge and does not perform
breast
self-examinations. Which of the following is the most
likely diagnosis?
A) Breast abscess
B) Breast cyst
C) Breast hematoma
D) Fibroadenoma

E) Fibrocystic changes of the breast


F) Intraductal adenoma
38. A 35-year-old woman is brought to the physician
by her husband because of increasing memory loss and involuntary
"dancing" movements over the past 6 weeks. She has had difficulty
remembering things, particularly recent events. She has had no
disturbances in sleep or appetite. Her medical history is unremarkable. She does
not
remember her mother who died 25 years ago, but her father told her that
her mother had similar symptoms several years before her death.
Neurologic examination shows involuntary choreiform movements. She has achildlik
e affect. She
describes her mood as good. On recall testing, she can remember one out
of three objects after 3 minutes with distraction; she
cannot remember the other objects with prompting. She cannot remember
the date of her marriage or her previous address. The most likely
cause of these symptoms is a lesion at which of the following locations?
A) Caudate nucleus
B) Parietal lobe
C) Prefrontal lobe
D) Putamen
E) Temporal lobe
39. A 67-year-old man comes to the physician
because of insomnia for 2 years. He goes to bed at 11:00 PM after taking a
bath but does not fall asleep until midnight. He usually wakes up twice
each night: once around 3:00 AM to void and again at 6:00 AM. He stays
in bed until his alarm goes off at 7 AM. He is concerned because he
used to sleep 8 hours daily. He recently retired from his job as an
attorney. He takes no medications. He drinks one beer with lunch 3 days
each week and one glass of wine with dinner each evening. He walks 1
mile daily after dinner. Physical examination shows no abnormalities.
His mood is neutral, and cognition is intact. Urine toxicology
screening is negative. Which of the following is the most likely cause of
this patient's symptoms?
A) Advanced sleep phase syndrome
B) Alcohol abuse
C) Delayed sleep phase syndrome
D) Major depressive disorder
E) Melatonin deficiency
F) Poor sleep hygiene
G) Normal aging

40. An asymptomatic 37-year-old woman comes for a


follow-up examination 1 year after receiving a renal transplant.
Current medications include felodipine, enalapril, cyclosporine,pravastatin, and
penicillin
G. She appears well. Her temperature is 36.8 C (98.2F), blood
pressure is 160/95 mm Hg, pulse is 80/min, and
respirations are 12/min. Examination shows no other abnormalities. Which of
the following medications is the most likely cause of this patient's hypertensio
n?
A) Cyclosporine
B) Enalapril
C) Felodipine
D) Penicillin G
E) Pravastatin
41. A 4080-g (9-lb) male newborn is delivered at
term to a 32-year-old woman, gravida 2, para 1. Apgar scores
are 8 and 9 at 1 and 5 minutes, respectively. Examination in the delivery
room shows fracture of the right clavicle. Which of the following is the
most likely sequela of this condition?
A) Correction only with casting
B) Correction only with physical therapy
C) Left-hand dominance
D) Permanent nerve damage
E) Spontaneous healing without treatment
42. A 22-year-old man with schizoaffective disorder
is brought to the emergency department 2 hours after the sudden
onset of neck and back pain. The symptoms began after taking one of his
medications for his psychiatric disorder. The dosage was increased
yesterday, but he does not recall the names of either of his medications. He
has no other history of serious illness. Physical examination
shows rigid contraction of the neck and back muscles with arching. On mental
status examination, he is alert and cooperative and hears a
faint voice that tells him to wash windows. He is oriented to person, place, and
time. Which of the following medications is most likely responsible for
this patient's symptoms?
A)
B)
C)
D)
E)

Clozapine
Haloperidol
Lithium carbonate
Trazodone
Valproic acid

43. A 52-year-old woman, gravida 3, para 3, comes


to the physician because of irregular vaginal bleeding over the past 2
months. She has hypertension treated with enalapril and type 2 diabetes mellitus

well
controlled with diet. Menopause occurred 2 years ago.
Her maternal aunt had breast cancer at the age of 70 years. The patient
weighs 88 kg (195 lb) and is 160 cm (63 in) tall. Examination
shows no other abnormalities. An endometrial biopsy specimen shows
adenocarcinoma. Which of the following is the most significant predisposing
factor for this patient's endometrial cancer?
A)
B)
C)
D)
E)

Heredity
Hypertension
Parity
Type 2 diabetes mellitus
Weight

44. A 67-year-old man is brought to the emergency


department 2 hours after the onset of weakness and double vision. He has
hypertension and hyperlipidemia treated with metoprolol, captopril, and
atorvastatin. His blood pressure is 190/106 mm Hg. Neurologic
examination shows left-sided facial weakness including the forehead.
There is palsy of left conjugate gaze, and the left eye fails to adduct on
right gaze.
Vertical eye movements are intact. Muscle strength is
3/5 in the right upper and lower extremities. Deep tendon reflexes are brisk, an
d Babinski's
sign is present on the right. Which of the following is the most likely location
of this patient's lesion?
A)
B)
C)
D)
E)

Bilateral thalamic
Left frontal
Left pontine
Right caudate
Right midbrain

45. Six weeks after spontaneous drainage of an anal


abscess, a 32-year-old man has persistent blood-stained purulent
fluid on his underwear. He has not had significant anal pain since
drainage of the boil. Bowel movements are normal. Which of the following is
the most likely diagnosis?
A)Anal fissure
B) Fistula in ano
C) Pruritus ani
D) Thrombosed external hemorrhoids
E) Thrombosed internal hemorrhoids
46. A previously healthy 14-year-old girl is
brought to the physician because of a 2-day history of fever and pain
and swelling of the right knee. She remembers injuring the knee while
playing soccer last week, but she was able to finish the game. She has nohistory
of rash or
joint pain. Her sister has inflammatory boweldisease. The patient's
temperature is 39 C (102.2 F), blood pressure is 110/80 mm Hg, pulse is 95/min,
and respirations are 20/min. Examination of the right knee shows swelling, tende
rness, warmth, and erythema;
range of motion is limited. Which of the following is the most
appropriate next step in management?
A) X-ray film of the right knee
B) Gastrointestinal series with small-bowel follow-through

C)
D)
E)
F)

Nuclear scan of the right knee


MRI of the right knee
Antibiotic therapy
Arthrocentesis

block 2
dolly123 - 11/07/06 17:58 #548028
1.A 25-year-old man is brought to the emergency
department after being
discovered semiconscious and incoherent at home. On
arrival, he is stuporous. His blood pressure is 105/70 mm Hg, pulse
is 80/min, and respirations are 12/min. Examination shows cool, damp
skin. The pupils are pinpoint and react sluggishly to light. Which of the
following is the most likely substance taken?
A)
B)
C)
D)
E)

Alcohol
Barbiturates
Cocaine
Heroin
LSD

2. A 15-year-old girl is brought to the emergency


department by her 20-year-old sister because of a 1-week history of
fatigue, nausea, and abdominal pain. Menarche was at the age of 12 years,
and her last menstrual period was 3 weeks ago. She has not had
vaginal discharge. She is sexually active, and she and her partner use
condoms inconsistently. Her temperature is 37 C (98.6 F), blood pressure is
110/60 mm Hg, and pulse is 95/min. Abdominal examination shows mild
bilateral lower quadrant tenderness. Before a pelvic examination and
a pregnancy test can be performed in this patient, consent must be obtained
from which of the following?
A) The court
B) The patient
C) The patient's parent
D) The patient's sister
E) No consent is necessary

3. A previously healthy 57-year-old man comes to


the physician because of a nonpruritic rash over both legs for 1
week and a low-grade fever for 2 days. He recently returned from a 2-week
canoe trip on a river in Minnesota. His temperature is 37.2 C (99 F).
Examination shows a 4 x 6-cm, macular, dark pink, ovoid lesion on the right post
erior hip
with central clearing and a punctate eschar near the
center. There are macular ring lesions with central clearing over the
medial tibia and anterior thighs of the lower extremities. He has no
lymphadenopathy. Which of the following is the most likely causal
organism?
A)
B)
C)
D)
E)

Borrelia burgdorferi
Brucella melitensis
Francisella tularensis
Leptospira interrogans
Rickettsia rickettsii

4. A 57-year-old woman comes to the physician


because of a 2-year history of increasing menstrual flow. She has not had
hot flashes, insomnia, or change in bowel or bladder function. Her
last menstrual period was 2 weeks ago. Pelvic examination shows a
normal-appearing vulva, vagina, and cervix. The uterus is consistent in size
with an 8-week gestation. Bimanual examination shows a 4-cm, firm,
nontender left ovary. An endometrial biopsy specimen shows atypical complex
endometrial hyperplasia. Which of the following is the most likely
cause of this patient's hyperplasia?
A) Adrenal adenoma
B) Brenner tumor
C) Carcinoid tumor
D) Granulosa cell tumor
E) Hyperthecosis
F) Islet cell tumor
G) Sertoli-Leydig cell tumor
5. A 67-year-old woman comes to the physician
because of vaginal bleeding for 10 days. She has been soaking one
sanitary pad daily. Menopause was 10 years ago. Her last Pap smear 5
years ago showed normal findings. Her temperature is 37 C (98.6 F), blood
pressure is 128/78 mmHg, pulse is 70/min, and respirations are 12/min.
Pelvic examination shows a normal-appearing cervix and a small amount of blood a
t the
cervical os. There is moderate thinning of the
vaginal mucosa. The uterus is irregular and consistent in size with a 10-week
gestation. There are no adnexal masses. Rectovaginal examination shows
no abnormalities. Test of the stool for occult blood is negative. Which
of the following is the most appropriate next step in diagnosis?
A)Transvaginal ultrasonography of the pelvis
B) Colposcopic-guided biopsy
C) Endometrial biopsy
D) Endometrial ablation
E) Culdocentesis
6. During a routine examination, a 32-year-old man
has a blood pressure of 120/80 mm Hg. He is concerned because his
father, grandfather, and two uncles have hypertension. He works as a
systems programmer for a large computer company and frequently has to meet
tight deadlines. He has smoked one pack of cigarettes daily for 10
years. He is 4.5 kg (10 lb) overweight and drinks three cups of coffee
daily. Which of the following measures is most likely to reduce this
patient's risk for hypertension over the next 5 years?
A) Increase intake of dietary fiber

B) Restrict caffeine
C) Stress management
D) Weight loss

7. A previously healthy 16-year-old boy comes to


the physician because of persistent pain in his left testicle for 24
hours. He has not had any penile discharge. There is no history of
trauma, but he plays soccer every day. Two months ago, he had sexual
intercourse for the first time, and he used a condom. Examination shows
an edematous, erythematous, exquisitely tender left scrotum that is
lower than the right. Elevating the left testicle relieves the pain. The
cremasteric reflex is present. Urinalysis shows 10 leukocytes/hpf and 1+
leukocyte esterase. A technetium 99m scan shows increased uptake in the
left testicle. Which of the following is the most likely cause of the pain?
A)
B)
C)
D)
E)

Cystitis
Epididymitis
Spermatocele
Testicular torsion
Testicular tumor

8. A 47-year-old woman comes to the physician


because of persistent nonproductive cough for 6 weeks. She has not had
fever or weight loss. She has hypertension treated with enalapril for the
past 3 months. She does not smoke. There is no history of lung disease.
She weighs 54 kg (120 lb) and is 163 cm (64 in) tall. Her temperature
is 37 C (98.6 F), blood pressure is 130/80 mm Hg, pulse is 70/min, and
respirations are 12/min. Examination and an x-ray film of the chest
show no abnormalities. Which of the following is the most
likely mechanism of this patient's cough?
A)
B)
C)
D)
E)

Decreased
Decreased
Increased
Increased
Increased

plasma renin activity


serum angiotensin II levels
serum angiotensin I levels
serum bradykinin levels
serum histamine levels

9. A 57-year-old man has been hospitalized for 2


days for treatment of unstable angina pectoris. He is currently
receiving intravenous heparin and undergoing evaluation for coronary artery
bypass grafting. His blood pressure is 160/90 mm Hg, pulse is 88/min,
and respirations are 16/min. Laboratory studies show:
Platelet count 90,000/mm3
Prothrombin time 12 sec (INR=1.1)
Partial thromboplastin time 35 sec
Which of the following is the most likely cause of
these findings?
A) Excessive platelet destruction

B)
C)
D)
E)

Factor VIII deficiency


Inadequate platelet production
Uncontrolled activation of coagulation and fibrinolytic cascades
Vitamin K deficiency

10. A study is conducted to assess the


effectiveness of a new blood test for early detection of prostate cancer. Ten
thousand healthy men over the age of 50 years are randomly assigned to
receive either annual rectal examination or annual screening with the new
blood test. After 5 years, results show that of the 50 men in the blood
test group that were diagnosed with prostate cancer, 40 were living 2years after
the
diagnosis was made. In comparison, only 15 out of 45
men in the rectal examination group survived 2 years after being
diagnosed with prostate cancer. Researchers conclude that the blood test
increases survival compared with rectal examination. Which of the
following potential flaws is most likely to invalidate this conclusion?
A)
B)
C)
D)
E)

Age of the patients


Diagnostic bias
Lead time bias
Recall bias
Type II error

11. A 62-year-old man comes to the emergency department because of


progressive shortness of breath for 3 days. He has
not had chest pain, orthopnea, or paroxysmal nocturnal dyspnea. He
completed chemotherapy for small cell carcinoma of the lung 10 months ago. He
has a history of twice nightly nocturia that has resolved over the past 3
days. He smoked two packs of cigarettes daily for 30 years but quit 1 year
ago. His blood pressure is 96/60 mm Hg, and pulse is 116/min. There
is jugular venous distention to the angle of the jaw. The lungs are clear to
auscultation. Cardiac examination shows distant heart
sounds, an S1 and S2, and no gallops or rubs. The liver has a span of 12 cm and
is
tender. There is no pedal edema. Laboratory studies show:
Hemoglobin 10 g/dL
Serum
Na+ 135 mEq/L
Cl 110 mEq/L
K+ 4.2 mEq/L
HCO3 22 mEq/L
Urea nitrogen (BUN) 40 mg/dL
Creatinine 1.6 mg/dL
An ECG shows diminished amplitude of the QRS
complexes. An x-ray film of the chest shows clear lung fields with an enlarged
cardiac silhouette. Which of the following findings is most
likely to be accentuated?
A)
B)
C)
D)
E)

Cardiac output
Fall in systolic arterial pressure with inspiration
Left ventricular end-diastolic pressure
Mitral regurgitation
Ventricular septal wall motion

12. A 35-year-old woman comes to the physician


because of two 12-hour episodes of dizziness over the past 3 months.
During episodes, she experiences the acute onset of rotatory vertigo and
imbalance, decreased hearing, tinnitus, a sense of fullness of the right
ear, and vomiting. Examination shows a mild hearing loss of the right
ear. Which of the following is the most likely diagnosis?
A)
B)
C)
D)
E)

Acoustic neuroma
Benign positional vertigo
Brain stem transient ischemic attacks
Meniere's disease
Viral labyrinthitis

13. An obese 33-year-old woman has had four 12-hour


episodes of severe, sharp, penetrating pain in the right upper quadrant of the a
bdomen
associated with vomiting but no fever. She has no
diarrhea, dysuria, or jaundice and is asymptomatic between episodes. There
is slight tenderness to deep palpation in the right upper
quadrant. Which of the following is the most appropriate next step in
diagnosis?
A)
B)
C)
D)
E)

Supine and erect x-ray films of the abdomen


Upper gastrointestinal series
Ultrasonography of the upper abdomen
CT scan of the abdomen
HIDA scan of the biliary tract

14. An otherwise healthy 19-year-old woman comes to


the physician because of a 3-year history of intermittent facial
blemishes. She drinks wine occasionally on weekends. She takes no
medications. Examination shows multiple 1- to 2-mm red and white papules and
larger red nodules on the forehead and cheeks. Which of the following is the mos
t
appropriate initial pharmacotherapy?
A)
B)
C)
D)
E)

Oral isotretinoin
Systemic corticosteroids
Topical benzoyl peroxide
Topical corticosteroids
Topical metronidazole

15. A previously healthy 67-year-old man comes to the physician


because of a 4-month history of hand weakness,
intermittent tingling of the small fingers of his hands, and mild neck pain.
Examination shows wasting, weakness, and fasciculations of the
interossei muscles. Sensation is decreased to pinprick and vibration in the smal
l

fingers of each hand. Triceps tendon reflexes are decreased. Which


of the following is the most likely diagnosis?
A)
B)
C)
D)
E)
F)
G)
H)
I)

Amyotrophic lateral sclerosis


Cervical spondylosis
Multiple sclerosis
Myasthenia gravis
Myasthenic (Lambert-Eaton) syndrome
Myotonic muscular dystrophy
Polymyalgia rheumatica
Polymyositis
Progressive neuropathic (peroneal) muscular atrophY

16. A 19-year-old college student comes to student


health services because of constant worrying since starting his
freshman year 6 months ago. He attends college several hundred miles away
from his hometown. He reports feeling constantly scrutinized by other
students and professors. He feels embarrassed and anxious in class and is in con
stant fear
of blushing. He has started to skip his classes
because of his anxiety and worries that he will not be able to complete the
school year. He drinks alcohol occasionally because it helps him
overcome his fear of being embarrassed in front of others. He does not use
illicit drugs. Physical examination shows no abnormalities. On
mental status examination, he is mildly anxious. He exhibits fair
eye contact and shifts uncomfortably in his chair. Laboratory studies are
within normal limits. Which of the following is the most appropriate
pharmacotherapy for this patient?
A)
B)
C)
D)
E)
F)

Disulfiram
Donepezil
Fluoxetine
Haloperidol
Methylphenidate
Valproic acid

17. A 42-year-old woman comes for a follow-up examination.


Two weeks ago, her blood pressure was 152/94 mm Hg during a routine
visit. Her blood pressure today is 150/94 mm Hg, pulse is 76/min, and
respirations are 14/min. Examination shows no other abnormalities.
Serum studies show:
Na+ 142 mEq/L
Cl 105 mEq/L
K+ 4 mEq/L
HCO3 26 mEq/L
Urea nitrogen (BUN) 12 mg/dL
Glucose 101 mg/dL
Creatinine 0.8 mg/dL
An ECG shows no abnormalities. Which of the following
is the most appropriate next step in management?

A)
B)
C)
D)
E)

Measurement of plasma renin activity


Serum lipid studies
24-Hour urine collection for measurement of metanephrine level
Echocardiography
Captopril renal scan

18. A 6-year-old boy with cystic fibrosis is


brought to the physician by his mother because his skin has been cool
and clammy for 30 minutes. Earlier in the day, he had been playing
outdoors, and the temperature was 99 F. When returning indoors, he was
thirsty and restless. His blood pressure is 70/40 mm Hg, and pulse is 120/min.
Examination shows dry mucous membranes. Serum sodium level is 128 mEq/L, and ser
um
chloride level is 87 mEq/L. Which of the following is
the most likely explanation for these findings?
A)
B)
C)
D)
E)

Excessive sweat electrolyte level


Excessive sweat volume
Excessive urinary output
Excessive vasopressor secretion
Inadequate sweat production

19. A 27-year-old primigravid woman at 29 weeks' gestation


comes to the emergency department because of a 24-hour history of
increasingly severe right-sided abdominal pain and no appetite. She has
vomited twice over the past 4 hours. She has not had vaginal bleeding.
Her temperature is 38.2 C (100.8 F). Examination shows exquisite
tenderness of the right lateral flank and the fundus. There are no
peritoneal signs. Bowel sounds are absent. The fetal heart rate is 144/min.
Laboratory studies show:
Hematocrit 37%
Leukocyte count 16,000/mm3
Serum
Total bilirubin 1.1 mg/dL
Amylase 32 U/L
Lactate dehydrogenase 110 U/L
Urine WBC 3 5/hpf
Which of the following is the most likely diagnosis?
A)
B)
C)
D)
E)

Abruptio placentae
Appendicitis
Cholelithiasis
Colitis
Pyelonephritis

20. A 32-year-old woman has had a lump in her neck


for 6 months. She has a 1.2-cm solitary left thyroid nodule.
Fine-needle aspiration cytology of the mass is consistent with a low-grade
papillary malignancy. Which of the following is the most appropriate next
step in management?
A)
B)
C)
D)
E)

Thyroid scan
131I therapy
Propylthiouracil therapy
Thyroxine therapy
Left thyroid lobectomy

For each patient with vaginal bleeding, select the


most likely diagnosis.
A)Bacterial vaginosis
B) Precocious puberty
C) Urinary tract infection
D) Vaginal foreign body
E) Vaginal laceration
F) von Willebrand's disease
21. A 12-year-old girl is brought to the physician
by her mother because of heavy vaginal bleeding since her first menstrual period
began 1
week ago. She has to change sanitary pads every 2
hours, and her mother is concerned that this is not normal. Her
blood pressure is 80/60 mm Hg, and pulse is 110/min. Breast and axillary and
pubic hair development are Tanner stage 4. Pelvic examination
shows normal external genitalia, a small normal-appearing cervix, and a
small uterus. Her hemoglobin level is 7 g/dL.

22. A 7-year-old girl is brought to the physician


by her mother 4 hours after she noticed that her daughter had blood on
her underwear. She has no signs of sexual development. Examination
shows normal external genitalia. There is a foul-smelling discharge.

23. A 10-year-old girl is brought to the physician because


of temperatures to 40 C (104 F) and headaches for 1 week. Her mother
says that fever and chills occur every other day and typically last
for several hours. She appears ill and is lethargic during febrile
episodes. She and her family returned from a trip to West Africa 2 weeks
ago. She currently appears ill. Her temperature is 40 C (104 F), blood
pressure is 94/64 mm Hg, pulse is 146/min, and respirations are 20/min.
Examination shows mild scleral icterus and conjunctival pallor. The
liver edge is palpated 3 cm below the right costal margin, and the
spleen tip is palpated 3 cm below the left costal margin. Laboratory studies
show:
Hemoglobin 8 g/dL

Leukocyte count 6400/mm3


Segmented neutrophils 46%
Eosinophils 5%
Lymphocytes 40%
Monocytes 9%
Serum
Na+ 132 mEq/L
Cl 98 mEq/L
K+ 4.2 mEq/L
HCO3 16 mEq/L
Urea nitrogen (BUN) 21 mg/dL
Bilirubin
Total 5.2 mg/dL
Direct 0.8 mg/dL
Aspartate aminotransferase (AST, GOT) 64 U/L
Alanine aminotransferase (ALT, GPT) 98 U/L
Urine Color tea
Blood strongly positive
RBC occasional
Which of the following measures is most likely to have
prevented this condition?
A)
B)
C)
D)
E)
F)

Hepatitis A vaccine
Typhoid vaccine
Oral isoniazid prophylaxis
Oral mefloquine prophylaxis
Oral trimethoprim-sulfamethoxazole prophylaxis
Intramuscular immune globulin

24. A 6-month-old boy is brought to the physician


because of seizures over the past week. His parents note that
during this period, he has had 10 to 20 episodes of throwing out his arms for 1
to 2 seconds then crying as if he were afraid of something. He was born
at term following an uncomplicated pregnancy and delivery.
Development is appropriate for age. Two days ago, treatment with amoxicillin was
begun for otitis media. He appears pale and apathetic. Examination
shows multiple white patches over the skin that become more distinct
with Wood's lamp examination. Neurologic examination shows no
abnormalities. An MRI of the brain is shown. Which of the following is the most
likely diagnosis?
A)
B)
C)
D)
E)

Congenital cytomegalovirus encephalopathy


Congenital toxoplasmosis encephalopathy
Hypoxic-ischemic encephalopathy
Neurofibromatosis
Tuberous sclerosis

25. An ELISA is used to detect Chlamydia trachomatis


infection in patients seen at a family planning clinic. In the first 500
patients, cultures are also done to check the accuracy of the ELISA. The
results are listed below:
Chlamydia culture
+
ELISA
+

38

455

Which of the following is the positive predictive


value of the ELISA?
A)
B)
C)
D)
E)

2/40
5/43
38/40
38/43
38/493

26. A 27-year-old woman is brought to the emergency


department by her mother who found her comatose 30 minutes ago. Her
mother says that her daughter had been having lower abdominal pain and
vaginal bleeding over the past week. The patient had an ectopic
pregnancy 2 years ago and was also treated with doxycycline for pelvic
inflammatory disease at that time. Her blood pressure is 40/20 mm Hg, pulse
is 160/min, and respirations are 24/min. The abdomen is distended and
rigid with decreased bowel sounds. Hemoglobin level is 4.2 g/dL,
and leukocyte count is 12,500/mm3. Culdocentesis is positive. Which of the
following is the most appropriate next step in management?
A)
B)
C)
D)
E)
F)
G)
H)
I)
J)

Bromocriptine therapy
Clomiphene therapy
Conjugated estrogen therapy
Ergot derivative therapy
Hysteroscopy
Laparoscopy
Dilatation and curettage
Endometrial ablation
Exploratory laparotomy
Total abdominal hysterectomy

27. A 43-year-old man comes to the emergency


department because of
fever, chills, malaise, cough, and pleuritic chest
pain for 3 days; the cough is productive of foul-smelling, purulent sputum.
He has alcoholism. His temperature is 39.2 C (102.6 F),
blood pressure is 110/70 mmHg, pulse is 120/min, and respirations are 14/min.
Hemoglobin level is 15 g/dL, and leukocyte count is 25,000/mm3. An ECG
shows normal findings. An x-ray film of the chest shows a 4-cm

cavity in the superior segment of the right lower lobe and an air-fluid
level. While awaiting results of sputum culture, which of the following is
the most appropriate next step in management?
A)
B)
C)
D)
E)

Observation
Broad-spectrum antibiotic therapy
Isoniazid therapy
Intravenous amphotericin B therapy
Tube thoracostomy

28. A 12-year-old girl with type 1 diabetes mellitus is


brought to the physician because of shortness of breath and fatigue
for 1 day. Since menarche began 4 months ago, she has had one episode
of diabetic ketoacidosis per month; prior to that she had been
stable. Her blood pressure is 110/70 mm Hg, pulse is 140/min, and respirations a
re
36/min. She appears to be moderately dehydrated. Laboratory
studies show:
Serum
Na+ 132 mEq/L
Cl 90 mEq/L
K+ 5.9 mEq/L
HCO3 6 mEq/L
Urea nitrogen (BUN) 48 mg/dL
Glucose 600 mg/dL
Creatinine 2.8 mg/dL
Urine
Glucose 4+
Ketones 3+
Protein 1+
Which of the following is the most likely underlying
cause of this patient's respiratory disorder?
A)
B)
C)
D)
E)

Acute interstitial nephritis


Acute renal failure
Diabetic nephropathy
Hyperinsulinemia
Increased plasma ketone level

29. A 28-year-old woman develops fatigue and


orthopnea 3 weeks after
the uncomplicated delivery of her first child. Her

blood pressure is 115/78 mm Hg, and pulse is 112/min. Crackles are


heard at both lung bases. The point of maximal impulse is laterally
displaced and diffuse; an S3 is present. There is 2+ pedal edema. An x-ray
film of the chest shows vascular cephalization of pulmonary vasculature
and Kerley B lines. Which of the following is the most appropriate next
step in management?
A)
B)
C)
D)
E)

Measurement of serum creatine kinase and lactate dehydrogenase activities


Echocardiography
Dipyridamole thallium scan
Ventilation-perfusion lung scans
Myocardial biopsy

30. A previously healthy 22-year-old woman comes to


the physician because of a paroxysmal cough for 2 weeks. She works
as an oncology nurse. Her temperature is 37 C (98.6 F). Examination
shows no abnormalities except for intermittent coughing spells
during the examination. An x-ray film of the chest is normal. Cultures of
nasopharyngeal secretions grow Bordetella pertussis. Which of the
following is the most appropriate pharmacotherapy for this patient?
A)
B)
C)
D)
E)

Amoxicillin
Amoxicillin-clavulanate
Cefprozil
Erythromycin
Penicillin G

31. A 5-year-old girl is brought to the physician


because of a 2-day
history of temperatures to 39.5 C (103.1 F) and pain
in the right side. She has had two episodes of vomiting during this
period but no diarrhea or symptoms of upper respiratory tract infection. She
has vesicoureteral reflux and a history of recurrent
urinary tract infections. She appears ill. Her temperature is 39.8 C (103.6 F),
blood pressure is 110/60 mm Hg, pulse is 150/min, and respirations are
25/min. Examination shows right-sided costovertebral angle tenderness. In
addition to obtaining urine cultures, which of the following is
the most appropriate next step in management?
A)
B)
C)
D)
E)

Intravenous pyelography
Renal ultrasonography
Voiding cystourethrography
Intramuscular antibiotic therapy and reexamination in 24 hours
Intravenous antibiotic therapy

32. A 10-year-old boy is brought to the physician


because of weakness and decreased appetite for 3 months. He
weighs 30 kg (66 lb) and is 142 cm (56 in) tall; he has had a 2.3-kg (5-lb) weig
ht
loss since his last examination 6 months ago. He appears thin. Deep
tendon reflexes are brisk. Chvostek's sign is positive. His serum
calcium level is 6.5 mg/dL, and serum intact parathyroid hormone level is

190 pg/mL (N=9 65). In addition to calcium supplementation,


which of the following is the most appropriate next step in management?
A)
B)
C)
D)
E)

Oral phosphate supplementation


Oral vitamin D supplementation
Intranasal calcitonin therapy
Parenteral bisphosphate therapy
Parenteral parathyroid hormone therapy

33. A healthy 8-year-old girl is brought to the


physician in July for a well-child examination. Her mother says that
her daughter is spending the summer at a nearby lake. Over the past
month, she has had two episodes of painful sunburn despite her mother's
efforts, including SPF 25 sunblock just before she goes swimming and urging
her to wear a hat and long-sleeved garments. The child takes no
medications. She has blond hair, blue eyes, and a fair complexion. The
mother seeks advice about preventing further sun damage to her child's
skin. Which of the following is the most appropriate recommendation?
A)
B)
C)
D)
E)
F)

Prohibit swimming on cloudless days


Apply the sunblock lotion 45 minutes before swimming
Change to a higher-level SPF lotion
Apply Burrow's solution compresses after each overexposure
Daily use of antioxidant vitamin supplement
Early treatment of any sun overexposure with topical corticosteroid

34. Two days after admission to the hospital because of a 3-day


history of slurred speech, double vision, and
dysphagia, a 24-year-old woman becomes quadriplegic and requires intubation and
mechanical ventilation. Her medical history is unremarkable.
One week ago, she attended a family picnic; several of her family members have h
ad
abdominal cramps and diarrhea since the picnic. Her temperature is 37C (98.6 F),
blood
pressure is 120/80 mm Hg, and pulse is 120/min.
Examination shows dry mucous membranes, large unreactive pupils,
ophthalmoplegia, and profound facial weakness. There is areflexia,
quadriplegia, and no movement of the palate and tongue. Sensation is normal.
Babinski's sign is absent. Which of the following is the most
appropriate pharmacotherapy?
A)
B)
C)
D)
E)

Antitoxin
Azathioprine
Interferon
Pyridostigmine
Riluzole

35. A 2325-g (5 lb 2 oz) male newborn is delivered


at 33 weeks' gestation; Apgar scores are 7 and 8 at 1 and 5
minutes, respectively. The 13-year-old mother had no prenatal care and did not
know how much weight she gained. During the pregnancy, the mother
smoked marijuana and took over-the-counter vitamins occasionally; she did
not drink alcohol and had no illness except for an upper respiratory
tract infection 4 months ago. She did not know she was pregnant until 2
weeks ago; her family is unaware of her condition. She has had one
sexual partner. During the hospital stay, the newborn and his mother have no
complications. The newborn is at greatest risk for morbidity and mortality from
which
of the following?
A)
B)
C)
D)
E)

Child abuse
Congenital syphilis
Hypocalcemia
Lead poisoning
Seizures

36. A 2-month-old girl is brought to the physician


because of a 2-week history of progressive difficulty breathing and
poor feeding. She has had rapid and labored breathing and sweating
during feedings. There is no history of fever or viral illness. Her
temperature is 37 C (98.6 F), blood pressure is 80/60 mm Hg, pulse is 130/min,
and respirations are 40/min. Bilateral crackles are heard at both lung
bases. A grade 4/6 holosystolic murmur is heard along the left
sternal border; the precordium is hyperdynamic. The liver edge is
palpated 4 cm below the right costal margin. An x-ray film of the chest shows
cardiomegaly and pulmonary congestion. Which of the following is the
most likely underlying mechanism for this child's condition?
A)
B)
C)
D)
E)

Decreased systemic vascular resistance


Increased pulmonary vascular resistance
Increased systemic vascular resistance
Intracardiac left-to-right shunt
Intracardiac right-to-left shunt

37. A 37-year-old man comes to the physician 6


months prior to traveling to sub-Saharan Africa for 1 year. He has no
history of hepatitis and has no high-risk behavior for hepatitis B. Which
of the following is the most appropriate recommendation to prevent
hepatitis during and after his trip?
A)
B)
C)
D)
E)

Immune globulin
Hepatitis B immune globulin (HBIG) only
Hepatitis B vaccine series only
Hepatitis B vaccine series and HBIG
Hepatitis B vaccine series and hepatitis A vaccine

38. A 67-year-old man comes to the physician because of a


2-month history of progressive shortness of breath. He has had a
4.5-kg (10-lb) weight loss over the past 4 months. He has not had chest
pain. He has congestive heart failure treated with furosemide,
digoxin, and enalapril. He has smoked two packs of cigarettes daily for 30 years
.
He appears alert and is in no acute distress. His temperature is
37.2 C (99 F), blood pressure is 140/85 mm Hg, pulse is 84/min, and
respirations are 18/min. Examination shows no jugular venous distention. There
is dullness to percussion, and breath sounds are decreased at the left base. Car
diac
examination shows a laterally displaced point of
maximal impulse, normal S1 and S2, and an S3 at the apex. There is 1+ edema
over the extremities. An x-ray film of the chest shows an
enlarged cardiac silhouette, left hilar fullness, and a moderate-sized left pleu
ral
effusion. Thoracentesis yields straw-colored fluid. Laboratory studies show:
Serum
Glucose
Protein
Lactate
Pleural

90 mg/dL
7 g/dL
dehydrogenase 300 U/L
fluid

pH 7.25
Glucose 75 mg/dL
Protein 4.5 g/dL
Lactate dehydrogenase 280 U/L
Leukocyte count 2000/mm3
Segmented neutrophils 15%
Lymphocytes 85%
A Gram's stain and acid-fast stains are negative for
any organisms. Which of the following is the most likely cause of
this patient's pleural effusion?
A)
B)
C)
D)
E)
F)

Bacterial pneumonia
Collagen vascular disease
Congestive heart failure
Malignancy
Pulmonary embolus with infarction
Viral pleuritis

39. A previously healthy 67-year-old woman is


admitted to the hospital because of a 2-week history of dark urine,
clay-colored stools, and increasing jaundice. She has had a 9-kg (20-lb)
weight loss over the past 2 months due to loss of appetite. She also has
had generalized itching that is most severe at night. She has not had
any abdominal pain. Examination shows no abnormalities except for
jaundice. Which of the following is the most likely diagnosis?
A)
B)
C)
D)
E)

Common bile duct stone


Drug-induced jaundice
Hemolytic jaundice
Pancreatic carcinoma
Viral hepatitis

40. A 2-year-old boy is brought to the physician


because of fever and listlessness for 12 hours. He has had recurrent

episodes of pneumonia and otitis media over the past year. Two
maternal uncles died of pneumonia in early childhood. One year ago, he was at
the 50th percentile for height and weight; he is currently at the 25th
percentile for height and 10th percentile for weight. He appears
ill. His temperature is 39 C (102.2 F), blood pressure is 60/40 mm Hg, pulse
is 160/min, and respirations are 36/min. Examination shows cool and
mottled extremities. A blood culture grows Streptococcus pneumoniae. Serum
IgE, IgG, and IgM levels are markedly decreased. Which of the following diagnost
ic
tests is most likely to be abnormal?
A)
B)
C)
D)
E)

Candidal skin test


Flow cytometry identification of B lymphocytes
Nitroblue tetrazolium test
T-lymphocyte receptor stimulation by concanavalin A
Total serum hemolytic complement assay

41. A 1-week-old newborn has had poor feeding,


vomiting, and progressive lethargy over the past 4 days. She was
born at term; pregnancy, labor, and delivery were uncomplicated, and she had no
congenital anomalies. She is being breast-fed. She has a
healthy 2-year-old brother; a sister died at 10 days of age after a full-term bi
rth.
Examination shows decreased muscle tone and poor responsiveness;reflexes are nor
mal.
Serum bicarbonate level is 8 mEq/L, pH is 7.15, and plasma ammonia
level is 10 times the upper limit of normal. Which of
the following is the most likely cause?
A)
B)
C)
D)
E)

Mitochondrial disorder
Mucopolysaccharidoses disorder
Organic acid metabolism disorder
Renal tubular acidosis
X-linked leukodystrophy

For each patient with an infection, select the most


likely causal
organism.
A)
B)
C)
D)
E)
F)
G)
H)

Chlamydia trachomatis
Clostridium botulinum
Clostridium tetani
Group A streptococcus
Group B streptococcus
Listeria monocytogenes
Neisseria gonorrhoeae
Streptococcus pneumonia

42. A premature 18-hour-old newborn is intubated and


mechanically ventilated because of progressive respiratory
distress. Labor was complicated by maternal fever and increased leukocyte count;
membranes ruptured 36 hours before delivery. His blood pressure and urine
output have decreased since birth. The newborn appears acutely
ill, and peripheral pulses are not palpable; the skin is pale, cool, and mottled
.
The liver edge is palpable 4 cm below the right costal margin. His
leukocyte count is 5000/mm3, platelet count is 48,000/mm3, and partial
thromboplastin time is 60 sec. An x-ray film of the chest shows diffuse,
bilateral, interstitial infiltrates.

43. A 6-year-old boy has been unable to walk for 2


days because of a sore right knee. Three weeks ago he had a sore throat
and fever that resolved within 2 days. He appears acutely ill. His
temperature is 39.2 C (102.5 F), and pulse is 120/min. A grade 2/6
pansystolic murmur is heard at the apex. The right knee is red, tender, and
swollen; any motion is painful. His leukocyte count is 15,000/mm3,
and erythrocyte sedimentation rate is 120 mm

44. A 70-year-old woman comes to the physician


because of fatigue and increasing difficulty in her daily functioning
over the past 2 months. During the day, she lies in bed for hours and
cries. She has had a 10.9-kg (24-lb) weight loss over the past 4 weeks,
eats only cookies, has lost interest in almost everything, and wishes to
kill herself. Her husband died 4 months ago. Physical examination and
laboratory studies show normal findings except for decreased serum
albumin and total protein levels. Which of the following is the most
likely diagnosis?
A)
B)
C)
D)
E)

Adjustment disorder with depressed mood


Bereavement
Bipolar disorder, depressed
Dysthymic disorder
Major depressive disorder

45. A 42-year-old woman comes to the physician


because of increasing low back pain for 2 days. She is a daily intravenous
drug user. She has a history of pyelonephritis, abscesses at
injection sites, and pelvic inflammatory disease. Her temperature is 39 C
(102.2 F), blood pressure is 130/70 mm Hg, pulse is 84/min, and
respirations are 20/min. Examination shows warm, dry skin, a supple neck, and
no jugular venous distention. There is tenderness over L4. Pelvic
examination shows mild erythema around the cervical os and scant discharge;
there is no adnexal or cervical motion tenderness. Her hematocrit is 30%,
leukocyte count is 10,600/mm3, and serum glucose level is 110 mg/dL.
Urinalysis is within normal limits. Which of the following is the

most appropriate next step in management?


A)
B)
C)
D)
E)

Echocardiography
Renal ultrasonography
MRI of the back
Colposcopy
Laparoscopy

46. A 13-year-old boy is brought to the physician


by his mother because of frequent headaches over the past 5 weeks.
He describes the headaches as dull pain across his forehead; they occur
four to five times weekly. Acetaminophen does not relieve the pain. His
mother states that occasionally he has nausea without vomiting. He
has schizophrenia well controlled with risperidone. Physical
examination shows no abnormalities. An MRI of the brain is most likely to
show which of the following?
A) Bilateral increased caudate nuclei
B) Decreased cerebellar volume
C) Hippocampal symmetry
D) Increased lateral ventricle size
E) Multiple white-matter hyperintensities
Report Abuse

* block 3:--dolly123 - 11/07/06 17:59 #548031

1. A 42-year-old woman, gravida 2, para 2, comes to


the physician because of a 3-month history of swelling of her legs
and mild abdominal pain and bloating. Abdominal examination shows no
abnormalities. Rectovaginal examination shows fullness in the right
adnexa. Transvaginal ultrasonography shows an irregular mass in the right

ovary with some solid components to a predominantly cystic lesion.


Her serum CA 125 level is 120 U/mL (N<35). Treatment with which of the
following is most likely to have prevented this patient's symptoms?
A)
B)
C)
D)
E)

Antiestrogens
Antiprogestationals
Medroxyprogesterone
Oral contraceptives
Ovulation-inducing drugs

2. A 15-year-old girl is brought to the physician 3


months after she had a blood pressure of 150/95 mm Hg at a routine
examination prior to participation in school sports. She is asymptomatic
and has no history of serious illness. Twelve months ago, she was
diagnosed with a urinary tract infection and treated with oral
trimethoprim-sulfamethoxazole.
She currently takes no medications. Subsequent blood
pressure measurements on three separate occasions since the
last visit have been: 155/94 mm Hg, 145/90 mm Hg, and 150/92 mm Hg. She is at th
e
50th percentile for height and 95th percentile for weight. Her blood
pressure today is 150/90 mm Hg confirmed by a second measurement, pulse
is 80/min, and respirations are 12/min. Examination shows no other
abnormalities. Her hematocrit is 40%. Urinalysis is within normal limits.
Cardiac and renal ultrasonography shows no abnormalities. Which
of the following is the most appropriate next step in management?
A)
B)
C)
D)
E)

Exercise and weight reduction program


Measurement of urine catecholamine levels
Measurement of urine corticosteroid levels
Captopril therapy
Hydrochlorothiazide therapy

3. Over the past 4 years, a 40-year-old woman has


had increasing episodes of loss of urine and difficulty emptying her
bladder. She has had no dysuria. She has a 30-year history of type 1
diabetes mellitus. She weighs 66 kg (145 lb) and is 175 cm (69 in) tall.
Pelvic examination shows a moderate cystocele. Postvoiding
catheterization yields 700 mL of clear urine. Which of the following is the most
likely cause of the patient's genitourinary symptoms?
A)
B)
C)
D)
E)

Carcinoma of the bladder


Detrusor instability
Neurogenic bladder
Urethral diverticulum
Uterine prolapse

4. A 57-year-old man comes to the emergency


department because of an episode of confusion, nervousness, sweating, and
palpitations 1 hour ago. He has had four similar episodes over the past 3
weeks; they last 2 to 3 minutes and are relieved by ingesting solid food
or liquids. His blood pressure is 140/74 mm Hg, pulse is 76/min, and
respirations are 18/min. Examination shows no other abnormalities.

The most appropriate next step in diagnosis is serum measurement of which


of the following?
A)
B)
C)
D)
E)

Fasting gastrin level


Fasting insulin and glucose levels
Glucagon level
Glucose and somatostatin level
Glucose and vasoactive intestinal

5. An otherwise healthy 15-year-old girl is brought


to the physician because she has never had a menstrual period. She reports that
breast
development started 1 year ago and pubic and axillary
hair development began 6 months ago. Examination shows normal
genitalia. Breast development is Tanner stage 4, and pubic hair
development is Tanner stage 3. Which of the following is the most appropriate ne
xt
step in management?
A) Reexamination in 1 year if the patient has not had menarche
B) Measurement of serum follicle-stimulating hormone and luteinizing hormone lev
els
C) Measurement of serum thyroid-stimulating hormone and prolactin levels
D) Karyotype analysis
E) Progesterone withdrawal test
F) Pelvic ultrasonographypolypeptide levels

6. A 2-month-old boy is brought to the physician because


of a 6-week history of persistent diarrhea and vomiting, most
pronounced after formula feedings. He has had a 113-g (4-oz) weight loss since
birth. He currently weighs 3100 g (6lb 13oz) and is 51 cm (20in) in length. He a

ppears irritable. Examination shows jaundice. The lungs are clear to


auscultation. No murmurs are heard. The liver is palpated 2 to 3 cm
below the right costal margin, and the spleen is palpated 1 to 2 cm below
the left costal margin. Laboratory studies show:
Serum
Glucose 35 mg/dL
Bilirubin (total) 2.3 mg/dL
Urine
Glucose negative
Reducing substances 3+
Which of the following is the most likely mechanism of
these findings?
A)
B)
C)
D)
E)
F)

Decreased gluconeogenesis
Decreased insulin secretion
Increased glucagon secretion
Increased gluconeogenesis
Increased insulin secretion
Insulin resistance

7. A 75-year-old man has had hypertension for 25years. There is an


unusually prominent pulsation of the abdominal aorta in the upper mid-abdomen. A
systolic bruit is heard at this site. Femoral, popliteal, and pedal pulses are
present. Which of the following is
the most appropriate initial diagnostic study?
A)
B)
C)
D)
E)

X-ray film of the abdomen


Abdominal ultrasonography
Doppler ultrasonography of the arteries of the legs
Abdominal aortography
Intravenous pyelography

8. A 14-year-old boy is brought to the physician


because of constant right knee pain for 2 weeks. The pain is not relieved

by rest or analgesics. There is no history of trauma. He is at


the 50th percentile for height and above the 95th percentile for weight.
He walks with a limp but is not in distress. His vital signs are
within normal limits. Examination shows full range of motion of both knees;
passive abduction and internal rotation of the right hip produce pain.
There is no swelling of either knee or hip. Neurologic
examination shows no abnormalities. Which of the following is the most likely di
agnosis?
A)
B)
C)
D)
E)

Juvenile rheumatoid arthritis


Osgood-Schlatter disease
Septic arthritis
Slipped capital femoral epiphysis
Toxic synovitis

9. A previously healthy 72-year-old man comes to the physician


because of a 2-year history of hand tremors and progressive difficulty
walking. He lives alone and has no close relatives.
He is alert and oriented. Physical examination shows a decreased rate
of eye blinking. Neurologic examination shows masked facies and a
pill-rolling resting tremor of both hands. There is cogwheel rigidity of
the upper extremities and generalized bradykinesia. His handwriting has
become small and illegible. He has a slow, shuffling, festinating gait with a te
ndency to
lean forward. Postural reflexes are impaired. This
patient is at greatest risk for injury due to which of the following?
A)
B)
C)
D)
E)

Bradykinesia
Cogwheel rigidity
Decreased rate of eye blinking
Postural reflex impairment
Tremor

10. A 47-year-old woman comes for a routine health


maintenance examination. She has a 10-year history of type 2
diabetes mellitus that is well controlled with an oral hypoglycemic agent. Her
mother died of a myocardial infarction at the age of 38 years. The
patient weighs 82 kg (180 lb) and is 163 cm (64 in) tall. Her blood
pressure is 150/95 mm Hg. Examination shows multiple small skin tags below
the eyebrows and on the nose and eyelids. Which of the following is
the most appropriate next step in diagnosis?
A)
B)
C)
D)
E)
F)

Measurement of serum follicle-stimulating hormone level


Serum lipid studies
24-Hour urine collection for measurement of creatinine clearance
X-ray film of the chest
ECG
No further studies indicated

11. A previously healthy 15-year-old boy is brought to the


physician because of a 5-day history of fever, intractable
nausea and vomiting, sore throat, and muscle pain. His mother has been giving
him ibuprofen and amoxicillin that was remaining from a previous
streptococcal throat infection. He appears ill, and his lips are parched.
His temperature is 38.9 C (102 F), blood pressure is 120/74 mm Hg while
supine and 100/70 mmHg while standing, and pulse is 92/min while supine
and 120/min while standing. Examination shows dry mucous membranes.
The oropharynx is erythematous without exudate. There is shotty
cervical adenopathy. The abdomen is soft without organomegaly. Laboratory
studies show:
Serum
Na+ 138 mEq/L
Cl 98 mEq/L
K+ 3.4 mEq/L
HCO3 21 mEq/L
Urea nitrogen (BUN) 55 mg/dL
Glucose 105 mg/dL
Creatinine 1.3 mg/dL
Amylase 40 U/L
Urine
Ketones moderate
WBC negative
RBC negative
Na+ 8 mEq/L
Protein negative
Which of the following is the most likely explanation
for this patient's renal insufficiency?
A)
B)
C)
D)
E)

Acute tubular necrosis


Amoxicillin-induced acute interstitial nephritis
Ibuprofen-induced renal failure
Post-streptococcal glomerulonephritis
Severe volume depletion

12. A 77-year-old woman is brought to the emergency


department after collapsing at home. Six hours ago, she had the sudden
onset of massive bright red rectal bleeding. On arrival, her blood
pressure is 90/60 mmHg, and pulse is 120/min. Abdominal examination shows
no abnormalities. Insertion of a nasogastric tube yields
clear aspirate. Her hematocrit is 28%. Which of the following is the most likely
diagnosis?
A)
B)
C)
D)
E)

Colon cancer
Diverticulosis
Duodenal ulcer
Hemorrhoids
Inflammatory bowel disease

13. A previously healthy 27-year-old woman comes to


the emergency department 1 hour after a 30-minute episode of
shortness of breath, nausea, diaphoresis, and crushing substernal chest
pain radiating to the left shoulder; the episode resolved spontaneously.
She is currently asymptomatic. Her blood pressure is 110/84 mm Hg,
pulse is 70/min, and respirations are 16/min. Examination shows no
abnormalities. An ECG shows a normal sinus rhythm and no other abnormalities. Te
n
minutes later, she sits up in bed stating that her symptoms have
returned. Her blood pressure now is 150/105 mm Hg, pulse is 120/min, and
respirations are 24/min. A repeat ECG shows 5-mm ST-segment elevation
in leads II, III, and aVF. Her symptoms resolve after administration of
aspirin and nitroglycerin. An angiography is ordered and shows no
evidence of coronary atherosclerosis. Which of the following is the most appropr
iate
pharmacotherapy?
A)
B)
C)
D)
E)
F)

Angiotensin-converting enzyme (ACE) inhibitor


Aspirin
Benzodiazepine
Calcium-channel blocking agent
Corticosteroids
Thiazide diuretic

Na+(mEq/L) Cl (mEq/L) K+(mEq/L) HCO3 (mEq/L) pH Specific gravity


A) 132
89
2.8
39
25

5.0

1.0

B) 133
15

110

3.9

16

6.0

1.0

C) 163
03

117

4.3

22

5.5

1.0

D) 165
30

115

4.5

19

5.0

1.0

14. An 8-week-old boy is brought to the physician by his mother


because of persistent vomiting for 2 days. He has spit up intermittently
since birth. He has had no diarrhea. He appears irritable. His
temperature is 38 C (100.4 F), blood pressure is 90/60mm Hg, pulse is
130/min, and respirations are 25/min. Examination
shows a sunken anterior fontanelle. Mucous membranes are dry. The lungs are
clear to auscultation. Heart sounds are normal. The abdomen is scaphoid, and bow
el sounds are active.

15. A 7-week-old boy is brought to the physician because of fever,


irritability, and lethargy for 3 days. He has had no vomiting,
diarrhea, or symptoms of an upper respiratory tract infection. He always seems
hungry to his mother despite being fed 3 ounces of formula every 3
hours. His mother also notes that he has many wet diapers throughout the
day and night. He currently appears irritable. His temperature is 37.8 C (100 F)
, blood pressure is 80/50 mm Hg, pulse is150/min, and
respirations are 25/min. Examination shows a sunken anterior fontanelle.
Mucous membranes are dry. There is tenting of the skin. Examination
shows no other abnormalities. Urinalysis is negative for glucose and protein.

16. A 57-year-old woman is extubated and transferred to


the recovery room after a cholecystectomy. She appears restless. Her
blood pressure is 120/70 mm Hg, pulse is 80/min, and respirations are 10/min. Ar
terial blood gas analysis on room air shows:
pH 7.24
PCO2 85 mm Hg
PO2 60 mm Hg

Intravenous naloxone therapy is begun, but she does not improve. Which
of the following is the most appropriate next step in management?
A)
B)
C)
D)
E)

Encouraging deep breathing and cough


Administration of 40% oxygen via nasal cannula
Administration of furosemide
Transfusion of 1 unit of packed red blood cells
Reintubation and mechanical ventilation

17. A previously healthy 24-year-old woman is brought to the


physician by her husband because of several episodes
of loss of consciousness over the past 4 days. Her husband reports that during
episodes, she jerks her arms and legs wildly. Each episode lasts up
to 1 hour; between episodes, her behavior is normal. She is planning to
move to another state because of her husband's work. She has been
extremely anxious and upset about the move because she will have to leave
her mother, who was recently diagnosed with breast cancer. There is no
family history of seizure disorder. Her temperature is 36.7 C (98 F),
blood pressure is 130/80 mm Hg, pulse is 84/min, and respirations are
18/min. Neurologic examination shows no abnormalities.
Electroencephalography shows normal findings during an episode of shaking. Which
of the following is the most likely underlying cause?
A)
B)
C)
D)
E)
F)

Catatonia
Complex partial seizure
Conversion reaction
Dissociative fugue
Malingering
Tonic-clonic seizure

18. A 37-year-old man comes to the physician


because of a 3-month history of increasing pain of his shoulders and upper
arms. Over the past 2 weeks, he has also had difficulty lifting heavy
objects off shelves in his garage. Two days ago, he burned his hand by
touching a hot pan and felt little discomfort. He sustained a concussion
in a motor vehicle collision at the age of 29 years. He has no
allergies. He takes no medications. He does not smoke cigarettes or use
illicit drugs, and he drinks one to two beers daily. Muscle strength is
decreased equally in both arms. Temperature and pain sensation are
decreased in both hands, but light touch is normal. Muscle strength and
sensation are normal in the lower extremities. Neurologic examination shows
no other abnormalities. Which of the following is the most likely diagnosis?
A)
B)
C)
D)
E)
F)
G)

Alcoholic peripheral neuropathy


Ankylosing spondylitis
Guillain-Barr syndrome
Herniated intervertebral disc
Multiple sclerosis
Polymyositis
Syringomyelia

19. A 62-year-old woman with ovarian cancer comes


to the emergency department because of fever for 2 days. Ten days ago,

she received chemotherapy with paclitaxel and carboplatin. She


feels tired but has not had nausea or vomiting. Her temperature is 39.5 C
(103.1 F), blood pressure is 100/60 mm Hg, and pulse is 115/min. The
lungs are clear to auscultation. Examination shows a soft, nontender
abdomen. Her hematocrit is 32%, leukocyte count is 800/mm3, and
platelet count is 105,000/mm3. Serum electrolyte levels are within
normal limits. Which of the following is the most appropriate next step in treat
ment?
A)
B)
C)
D)
E)
F)

Plasmapheresis
Additional chemotherapy
Intravenous antibiotic therapy
Intravenous corticosteroid therapy
Transfusion of 2 units of leukocytes
Transfusion of 2 units of packed red blood cells

20. A 47-year-old woman who is visiting from Australia


comes to the physician because of increasing urine output over the
past month. She has had no dysuria or hematuria. She has a history of chronic he
adaches,
peptic ulcer disease, and urinary tract infections.
An evaluation 18 months ago for headaches, including CT scan of the head,
showed no abnormalities; treatment with ibuprofen and phenacetin
was initiated at that time, and her headaches have been well controlled. Her tem
perature is 37.1 C
(98.8 F), blood pressure is 140/82 mm Hg, pulse is78/min, and
respirations are 14/min. Examination shows no abnormalities. Laboratory studies
show:
Hematocrit 32%
Mean corpuscular volume 88 m3
Serum
Glucose 130 mg/dL
Creatinine 1.7 mg/dL
Urine
Protein 2+
WBC 8 10/hpf
RBC none
Bacteria none
Nitrates none
Test of the stool for occult blood is negative. Which
of the following
is the most appropriate next step in management?
A)
B)
C)
D)
E)

Intravenous pyelography
Discontinue current medication
Antibiotic therapy for recurrent urinary tract infections
Insulin therapy for diabetes mellitus
Upper endoscopy

20. A 47-year-old woman who is visiting from Australia


comes to the physician because of increasing urine output over the
past month. She has had no dysuria or hematuria. She has a history of chronic
headaches, peptic ulcer disease, and urinary tract infections. An evaluation 18
months
ago for headaches, including CT scan of the head,showed no
abnormalities; treatment with ibuprofen and phenacetin was initiated at that tim
e,
and her headaches have been well controlled. Her temperature is 37.1 C
(98.8 F), blood pressure is 140/82 mm Hg, pulse is78/min, and
respirations are 14/min. Examination shows no abnormalities. Laboratory studies
show:
Hematocrit 32%
Mean corpuscular volume 88 m3
Serum
Glucose 130 mg/dL
Creatinine 1.7 mg/dL
Urine
Protein 2+
WBC 8 10/hpf
RBC none
Bacteria none
Nitrates none
Test of the stool for occult blood is negative. Which
of the following
is the most appropriate next step in management?
A)
B)
C)
D)
E)

Intravenous pyelography
Discontinue current medication
Antibiotic therapy for recurrent urinary tract infections
Insulin therapy for diabetes mellitus
Upper endoscopy

21. A previously healthy 85-year-old man has had


abdominal distention, decreased caliber of stools, and decreased
appetite over the past 2 weeks and a 9-kg (20-lb) weight loss over the past 3
months. On sigmoidoscopy, he is found to have a constricting
adenocarcinoma of the sigmoid colon; imaging studies show three 1-cm
metastases to the liver. Which of the following is the most appropriate next
step in management?
A)
B)
C)
D)
E)

No treatment
Radiation therapy
Chemotherapy
Combination radiation therapy and chemotherapy
Resection of the colon tumor

22. An 8-year-old girl with asthma is brought to the physician 1 week after an a
cute exacerbation treated with a 5-day taper course of oral prednisone. This was
her first asthma attack of the
fall season. Medications include an inhaled corticosteroid daily
and a bronchodilator metered-dose inhaler as needed. Her last
immunizations were at the age of 5 years prior to entering kindergarten. Her
temperature is 37 C (98.6 F), pulse is 92/min, and respirations are 28/min. Exam
ination shows
end-expiratory wheezing with forced expiration. Administration of
which of the following vaccines is most appropriate at this visit?
A)
B)
C)
D)
E)

Haemophilus influenzae type b


Influenza virus
Meningococcal
23-Valent pneumococcal
Varicella

23. A 27-year-old man is brought to the hospital by


family members because he has remained in his room for 3 days. He
has refused to go to work or eat with the family, and he expresses concern
that family or friends may try to kill him. One week ago, he was
despondent when his girlfriend of 5 years abruptly ended their
relationship. He has no history of psychiatric illness. Which of the following
is the most likely diagnosis?
A)
B)
C)
D)
E)

Adjustment disorder with depressed mood


Bipolar disorder
Brief psychotic disorder
Dysthymic disorder
Schizoaffective disorder

24. A 35-year-old primigravid woman at 20 weeks'


gestation comes to the physician because of vaginal pressure and a
watery, pink vaginal discharge for 1 day. Her temperature is 37.5 C (99.5
F). The uterus is palpated at the umbilicus. Fetal heart rate is
140/min. Speculum examination shows that the upper vagina is filled with
bulging, shiny, smooth membranes. The cervix cannot be palpated.
Which of the following is the most likely mechanism for these findings?
A)
B)
C)
D)
E)

Abruptio placentae
Cervical incompetence
Premature labor
Uterine anomaly
Uterine infection

25. A previously healthy 45-year-old woman has had fever and


progressive confusion over the past 2 days. She is
now unable to perform activities of daily living. Her temperature is 38 C
(100.4 F). She is oriented only to person. There is no rash, and the
neck is supple. A CT scan of the head shows normal findings. Cerebrospinal
fluid analysis shows:
Leukocyte count 20/mm3
Glucose 45 mg/dL
Protein 110 mg/dL
Erythrocyte count
1000/mm3
Which of the following is the most likely diagnosis?
A) Bacterial meningitis
B) Acute alcohol intoxication
C) Brain stem infarction
D) Cerebral infarction
E) Cryptococcal meningitis
F) Enterovirus infection
G) Hepatic encephalopathy
H) Herpes simplex encephalitis

I) Huntington's disease
J) Hypoglycemia
K) Lyme disease

26. A 57-year-old man with multiple myeloma comes


to the physician because of a 12-hour history of fever, sharp chest
pain with deep inspiration, and cough productive of blood-tinged
sputum. His temperature is 38.3 C (101 F), blood pressure is 120/78 mm Hg, pulse
is 112/min, and respirations are 28/min. Crackles are heard at the
right lung base. His hemoglobin level is 9.2 g/dL, leukocyte count is
2600/mm3, and platelet count is 96,000/mm3. Empiric antibiotics
should be directed against which of the following organisms?
A) Listeria monocytogenes
B) Neisseria meningitidis
C) Pseudomonas aeruginosa
D) Streptococcus bovis
E) Streptococcus pneumoniae

27. A 19-year-old primigravid woman is brought to


the emergency department because of a 4-hour history of heavy
vaginal bleeding. She has vomited daily for the past month. Her last menstrual
period was 15 weeks ago. She has not received prenatal care. She
takes no medications. Her temperature is 37 C (98.6 F), blood pressure is
140/90 mm Hg, pulse is 80/min, and respirations are 20/min. Abdominal
examination shows a uterus consistent in size with a 20-week gestation
with no adnexal masses or tenderness. There is pedal edema. A serum
pregnancy test is positive. Urinalysis shows 1+ protein. Which of the
following is the most likely cause of this patient's vaginal bleeding?
A) Abruptio placentae
B) Ectopic pregnancy
C) Hydatidiform mole
D) Hyperthyroidism

E) Preeclampsia

28. A 40-year-old man has the sudden onset of


excruciating head and neck pain while carrying books from the basement to
the attic. His temperature is 37 C (98.6 F), blood pressure is 130/90 mm Hg, and
pulse is
90/min. He has photophobia and develops eye pain with lateral eye
movements. His neck is markedly stiff and cannot be passively flexed. He
has diffuse hyperreflexia in all extremities with normal strength and
sensation. Plantar reflexes are flexor bilaterally. Which of the
following is the most likely diagnosis?
A) Cervical osteoarthritis
B) Meningitis
C) Ruptured cervical disc
D) Subarachnoid hemorrhage
E) Syringomyelia

29. A 59-year-old woman comes to the emergency


department because of shortness of breath for 2 days. She had stage II
breast cancer 5 years ago treated with lumpectomy, radiation, and
chemotherapy. Her temperature is 37.5 C (99.5 F), blood pressure is
90/60 mm Hg, and respirations are 24/min. Examination shows jugular venous
distention. Heart sounds are distant. Which of the following is the most
appropriate next step in management?
A) Antibiotic therapy
B) Anticoagulant therapy
C) Intravenous digoxin therapy
D) Intravenous furosemide therapy
E) Chemotherapy
F) Radiation therapy
G) Pericardiocentesis

30. A 57-year-old man comes for a routine follow-up


examination. He has a 10-year history of an intermittent facial rash.

He has been taking propranolol for 2 months for hypertension.


Examination shows several erythematous pustules and papules involving
the nose and central face. There are telangiectasias at the base of the
papules. Which of the following is the most likely explanation for these
findings?
A) Acne rosacea
B) Acne vulgaris
C) Basal cell carcinoma
D) Discoid lupus erythematosus
E) Seborrheic dermatitis

31. A 42-year-old man comes to the physician


because of malaise, muscle and joint pain, and temperatures to 38.4 C
(101.1 F) for 3 days. Three months ago, he underwent cadaveric renal
transplantation resulting in immediate kidney function. At the time of
discharge, his serum creatinine level was 0.8 mg/dL. He is receiving
cyclosporine and corticosteroids. Examination shows no abnormalities.
His leukocyte count is 2700/mm3, and serum creatinine level is 1.6 mg/dL;
serum cyclosporine level is in the therapeutic range. A biopsy of the
transplanted kidney shows intracellular inclusion bodies. Which of the
following is the most appropriate next step in management?
A) Increase the dosage of corticosteroids
B) Increase the dosage of cyclosporine
C) Begin amphotericin therapy
D) Begin ganciclovir therapy
E) Begin heparin therapy

32. A 5-month-old boy is brought to the physician because of a


24-hour history of fever, cough, noisy breathing, and difficulty feeding.
His symptoms began 3 days ago with nasal discharge,
mild cough, and chest congestion. He appears somewhat irritable and
is crying. His temperature is 38.5 C (101.3 F), pulse is 108/min, and
respirations are 32/min and shallow with a prolonged expiratory phase.
On examination, the throat appears normal. A few small anterior and
posterior cervical nodes are palpable. Both eardrums are pink but have
normal landmarks and mobility. There is good air entry with diffuse
bilateral expiratory wheezes on auscultation. An x-ray film of the chest
shows hyperinflation. Which of the following is the most likely pathogen?
A) Adenovirus
B) Haemophilus influenzae
C) Mycoplasma pneumoniae
D) Respiratory syncytial virus
E) Streptococcus pneumoniae

33. Three days after undergoing a right hip replacement for


rheumatoid arthritis, a 77-year-old man is brought to
the physician because of a 2-day history of pain, burning, and itching of his
left eye and left side of his forehead. He has the sensation that there
is a speck of dirt in his left eye. Current medications include
prednisone and methotrexate. Examination of the left eye shows
conjunctival injection and swelling of the upper eyelid. There is an
erythematous rash over the left side of the forehead and tenderness to palpation
from
the upper eyelid to the vertex. A photograph of the rash is shown.
Which of the following is the most appropriate next step in management?
A) Measurement of erythrocyte sedimentation rate
B) MRI of the brain with contrast
C) Acyclovir therapy
D) Corticosteroid therapy
E) Lumbar puncture

34. A 54-year-old man with chronic obstructive pulmonary disease


undergoes a total hip arthroplasty for avascular necrosis of the femoral
head. On the second postoperative day, he has diffuse, profound
weakness and vomiting. His blood pressure is 85/50 mmHg, and pulse is
100/min. The operative site is clean and dry, with minimal output from the
drains. Hemoglobin level is 13.8 g/dL, serum sodium level is 132 mEq/L,
and serum potassium level is 5.8 mEq/L. Which of the following is most
likely to confirm the diagnosis?
A) Measurement of serum thyroid-stimulating hormone level
B) Direct antiglobulin (Coombs') test
C) ACTH stimulation test
D) Ventilation-perfusion lung scans
E) Echocardiography

35. A previously healthy 47-year-old nulliparous woman is brought to


the emergency department by ambulance because of acute
low back pain radiating to the right posterior leg for 2 hours. The
pain began when she bent over at work to retrieve a file from the
lowest drawer of a filing cabinet. She does not smoke cigarettes or
drink alcohol. Examination shows right paraspinous muscle spasm and
pain in the lower back with right straight-leg raising at 30 degrees. She says
that she plans to file a claim for a work-related injury. Which of the
following findings is the strongest risk factor for a prolonged episode
of pain in this patient?
A) Arrival for care in an ambulance
B) Claim that pain is work-related
C) Gender
D) Nulliparity
E) Positive straight-leg raising test
F) Radiation of the pain into the posterior lower
extremity

36. A healthy 27-year-old woman comes to the physician for an annual


examination. She is concerned about her risk for an abnormal Pap smear
in the future. A history of use of which of the following would
increase her risk for cervical cancer?
A) Alcohol
B) Cervical cap
C) Cigarettes
D) Isotretinoin
E) IUD

37. A 72-year-old man is brought to the physician by his daughter


because of painless jaundice for 1 month. His wife
died 10 years ago, and his daughter is his only child. Before examining
the patient, the daughter asks to speak privately with the physician
and asks that she be given the results of any tests. She specifically
requests that he not be given any "bad news." The patient is alert. His
vital signs are within normal limits. Examination shows scleral
icterus and jaundice. There is mild abdominal tenderness on palpation. He
is oriented to person, place, and time. A CT scan of the abdomen
shows a pancreatic mass with bile duct obstruction and probable metastatic
lesions in the liver. Which of the following is the most appropriate
next step?
A) Abide by the daughter's wishes
B) Ask the patient if he wishes to discuss his test results, preferably with his
daughter present
C) Tell the daughter it is a legal requirement to tell the patient any and all r
esults of medical testing
D) Consult with the hospital attorney
E) Ask another physician to take over the patient's care

38. A 66-year-old man has had numbness and tingling in the hands and
feet for 2 weeks. He lives in a homeless shelter and is well fed. He
has been treated for pulmonary tuberculosis for 4 months with
isoniazid, rifampin, ethambutol, and pyrazinamide. He is compliant with his
medication regimen but continues to abuse alcohol. His temperature is 37
C (98.6 F), blood pressure is 136/76 mm Hg, pulse is 72/min, and
respirations are 20/min. He is well nourished but depressed and irritable.
There is decreased sensation to pain and touch in the hands and feet in
a stocking-glove distribution. Which of the following is the most
likely nutritional deficiency?
A) Folic acid
B) Niacin
C) Vitamin A
D) Vitamin B1 (thiamine)
E) Vitamin B2 (riboflavin)
F) Vitamin B6
G) Vitamin B12 (cyanocobalamin)
H) Vitamin C
I) Vitamin D
J) Vitamin E
K) Vitamin K

39. A 63-year-old man is brought to the emergency department 3 hours


after the acute onset of severe right-sided flank pain. He has a
9-year history of gout. His blood pressure is 110/84mm Hg, pulse is
78/min, and respirations are 16/min. Examination
shows normal bowel sounds and no abdominal tenderness or masses. Urinalysis
shows 40 erythrocytes/hpf. Intravenous pyelography confirms a
right ureteral calculus. Which of the following is the most likely underlying
mechanism of this patient's urolithiasis?
A) An increase in urinary pH
B) Damage to the epithelial lining of the ureters
C) Lack of inhibitors of crystal formation
D) Presence of urease-splitting bacteria
E) Urinary supersaturation with uric acid

40. A 27-year-old nulligravid woman comes to the


physician for preconceptional counseling. She has a mechanical
mitral heart valve and chronic rheumatoid arthritis. Her cardiac status is
New York Heart Association Class II. She feels well. Current daily
medications include warfarin, prednisone, and acetaminophen with codeine.
Examination shows no abnormalities except for audible clicking from the
heart valve. Which of the following is the most appropriate advice
for this patient?
A) Chemical dependency counseling before pregnancy
B) Discontinuation of anticoagulant therapy during pregnancy
C) Discontinuation of prednisone during pregnancy
D) Switching from warfarin to heparin before pregnancy
E) No change in treatment before or during pregnancy

41. A 60-year-old man comes to the physician because of difficulty


sleeping and concentrating and a 5-kg (10-lb) weight loss over the past
3 months. He also has become withdrawn. He has had chronic pain since
sustaining fractures of the left lower extremity,pelvis, and several
ribs in a motor vehicle collision 2 years ago. He has a previous
history of alcohol abuse. He takes a nonsteroidal anti-inflammatory drug.
Which of the following is the most appropriate pharmacotherapy?
A) Carbamazepine
B) Chlordiazepoxide
C) Disulfiram
D) Lithium carbonate
E) Nortriptyline

42. An 82-year-old man comes to the physician because of a 3-day


history of low back pain that radiates to the right leg. He also has had
a lesion over the right shin and weakness of the right foot. He began
taking prednisone 2 weeks ago for acute bronchitis. He has chronic
obstructive pulmonary disease, benign prostatic hypertrophy, and glaucoma.
Examination shows numerous papular and vesicular
lesions over the right anterior and posterior shin. There is weakness of
right knee flexion, ankle dorsiflexion, plantar flexion, eversion, and
inversion; the right ankle reflex is absent. Sensation to pinprick and
cold is decreased over the right lower extremity. Which of the
following is the most likely causal organism?
A) Borrelia burgdorferi
B) Epstein-Barr virus
C) Herpes simplex virus 1
D) Poliovirus
E) Treponema pallidum
F) Varicella-zoster virus

43. A 30-year-old woman comes to the physician because of


long-standing unhappiness that may have started when she was rejected by her
classmates as a teenager. She says that although she has good days, many
days are dominated by negative thoughts about herself. She appears
somewhat sad and tends to be readily critical of herself. Although she
sleeps satisfactorily, she often finds her energy level decreased by the
end of the day. She also has been forgetful. She weighs 59 kg (130 lb)
and is 157 cm (62 in) tall. Physical examination and laboratory
studies show no abnormalities. Which of the following is the most likely
diagnosis?
A) Adjustment disorder with depressed mood
B) Depersonalization disorder
C) Dissociative identity disorder
D) Dysthymic disorder

E) Hypothyroidism
F) Major depressive disorder
G) Schizoaffective disorder
For each patient with cough, select the most appropriate next step in diagnosis.
A)
B)
C)
D)
E)

Measurement of serum a1-antitrypsin level


Methacholine challenge test
Quantitative measurement of serum antibody levels
Sweat chloride test
Ventilation-perfusion lung scans

44. A 27-year-old woman comes to the physician because of


intermittent episodes of shortness of breath and cough over the past 4 months.
She says that cold weather and exercise can precipitate her symptoms.
Examination shows no abnormalities. The lungs are clear to auscultation
and percussion. An x-ray film of the chest and spirometry show no abnormalities
.

45. A 32-year-old woman comes to the physician because of a 4-month history


of fatigue, cough, and shortness of breath with exertion. She has had
two episodes of pneumonia and one episode of severe sinusitis over the
past 2 years. She has never smoked. She takes no medications.
Crackles are heard at the left lung base. An x-ray film of the chest shows a
left lower lobe infiltrate and scarring of the right base.

46. A 32-year-old man is brought to the emergency department 30


minutes after being involved in a motor vehicle collision. He was the
restrained driver. On arrival, he is alert and has shortness of breath.
His blood pressure is 80/50 mm Hg, pulse is 130/min, and respirations
are 30/min. Examination shows jugular venous distention and abrasions
over the left hemithorax. The trachea is deviated to
the right. Breath sounds are absent on the left. Which of the following
is the most likely cause of the hypotension?
A) Cardiogenic shock
B) Congestive heart failure
C) Decreased systemic vascular resistance
D) Decreased venous return

E) Hypovolemic hypoperfusion
F) Increased systemic vascular resistance

BLOCK 4:-1. A previously healthy 2-year-old boy is brought to the physician


20 minutes after an episode of cyanosis and loss of consciousness that lasted 3
minutes. The symptoms occurred after his mother scolded him for climbing onto th
e dining room table. The mother says that the child began to cry, let out a deep
sigh, stopped breathing,
and jerked his arms and legs back and forth. On arrival, he is alert and active.
Neurologic examination shows no focal findings. Which of the following is
the most appropriate next step in management?
A) Reassurance
B) Electroencephalography

C) CT scan of the head


D) Anticonvulsant therapy
E) Lumbar puncture

2. A case-control study is conducted to assess the risk for


intussusception in infants under the age of 1 year who receive the rotavirus
vaccine. The medical records of all those who received the vaccine and
those who did not receive the vaccine over a 6-month period are
reviewed. Results show 125 cases per 100,000 infant-years for infants who
received the vaccine compared to 45 cases per 100,000 infant-years for
infants who did not receive the vaccine. The investigators conclude that
the relative risk for intussusception is 1.9 times greater in infants
who receive the rotavirus vaccine (95% confidence interval of 0.5 7.7 and
p=0.39). Which of the following is the most accurate interpretation of
these results?
A) The results do not show an association between rotavirus vaccine
and intussusception, but they may be related
B) The results show sufficient statistical power to identify an association betw
een rotavirus vaccine and intussusception
C) Rotavirus vaccine is associated with a 39% risk for intussusception
D) Rotavirus vaccine causes intussusception in 1.9% of infants
E) Rotavirus vaccine prevents 80 cases of intussusception per 100,000 infant-yea
rs

3. A 42-year-old woman, gravida 3, para 3, comes to the


emergency department 24 hours after the onset of moderate
epigastric pain radiating to the back. Her last menstrual period was 3 weeks ago
.
She has no history of serious illness and has never undergone an
operative procedure. She weighs 72 kg (160 lb) and is 157 cm (62 in) tall. Her
temperature is 37.2 C (99 F), blood pressure is 130/90 mm Hg, and
pulse is 100/min. Abdominal examination shows mild distention,
epigastric tenderness, and voluntary guarding. Test of the stool for occult
blood is negative. Laboratory studies show:

Hematocrit 44%
Leukocyte count 12,000/mm3
Serum
Na+ 138 mEq/L
Cl 100 mEq/L
K+ 4 mEq/L
HCO3 25 mEq/L
Bilirubin, total 1.6 mg/dL
Alkaline phosphatase 100 U/L
Aspartate aminotransferase (AST, GOT) 14 U/L
Alanine aminotransferase (ALT, GPT) 12 U/L
Amylase 1100 U/L
Ultrasonography shows gallstones; the gallbladder wall is 1 mm and the
common bile duct is 5 mm in diameter. Which of the
following is the most likely diagnosis?
A) Acute cholecystitis
B) Acute pancreatitis
C) Acute perihepatitis
D) Ascending cholangitis
E) Duodenal ulcer
F) Viral hepatitis
4. A 13-year-old girl is brought to the emergency
department because of shortness of breath for 2 hours. The symptoms
began after consuming chili, cornbread, and fruit salad with strawberries,
kiwi, and bananas. She has a 1-year history of shortness of breath while
playing soccer or baseball and uses a bronchodilator inhaler as needed while exe
rcising.
She is allergic to penicillin and pineapples. Her blood pressure is
80/60 mm Hg, pulse is 120/min and regular, and
respirations are 20/min with use of accessory muscles. Examination of the
lungs shows poor air entry bilaterally with diffuse expiratory wheezes.
Which of the following is the most appropriate initial pharmacotherapy?
A) Inhaled bronchodilators
B) Inhaled cromolyn sodium
C) Inhaled ipratropium bromide
D) Intravenous corticosteroids
E) Subcutaneous epinephrine
5. A 52-year-old woman comes to the emergency department
6 days after knee arthroplasty because of constant, right-sided chest
pain and shortness of breath for 24 hours. Her blood pressure is 110/50mm Hg, pu
lse is 114/min, and respirations are 24/min. Examination of the heart, lungs,
and extremities shows no abnormalities. Arterial blood gas analysis on
room air shows:
pH 7.49

PCO2 29 mm Hg
PO2 66 mm Hg
Ventilation-perfusion lung scans show a low probability for pulmonary
embolus. An ECG shows sinus tachycardia; an x-ray film of the chest
shows no abnormalities. After the evaluation, the patient is pain-free
and wishes to go home. Which of the following is the most appropriate
next step in management?
A) Discharge home and reexamination in 2 weeks
B) Exercise stress test
C) Pulmonary function tests
D) Echocardiography
E) Pulmonary angiography
F) Ibuprofen therapy

6. A 25-year-old woman comes to the physician because of "spells"


characterized by sweating, palpitations, and shortness
of breath that have awakened her at night several times over the past
3 months. She resigned from her job as a sales clerk 6 months ago
and now works from home as a telemarketer. She recently began going to the
grocery store late at night because she is too nervous around people.
She says that she has been feeling sad lately. On mental status
examination, she is fully oriented, and her range of affect is full. Which of
the following is the most likely diagnosis?
A) Adjustment disorder with anxiety
B) Adjustment disorder with depressed mood
C) Dysthymic disorder
D) Generalized anxiety disorder
E) Major depressive disorder
F) Panic disorder with agoraphobia
G) Post-traumatic stress disorder

7. A 65-year-old woman has a 6-month history of


progressive irritability, palpitations, heat intolerance, frequent
bowel movements, and a 6.8-kg (15-lb) weight loss. She has had a neck mass
for more than 10 years. 131I scan shows an enlarged thyroid gland with
multiple areas of increased and decreased uptake. Which of the
following is the most likely diagnosis?

A) Defect in thyroxine (T4) biosynthesis


B) Graves' disease
C) Multinodular goiter
D) Riedel's thyroiditis
E) Thyroid carcinoma
F) Thyroiditis
G) Toxic adenoma
H) Triiodothyronine (T3) thyrotoxicosis

A)Acute gastrointestinal bleeding


B) Adrenal insufficiency
C) Aortic valve rupture
D) Cardiac tamponade
E) Congestive heart failure

F) Pneumonia
G) Pulmonary embolism
H) Sepsis
8. A previously healthy 62-year-old man is brought
to the emergency department by paramedics 40 minutes after the sudden
onset of severe shortness of breath while dressing this morning. He
is unable to provide additional medical history. He is in severe
respiratory distress. His temperature is 37.8 C (100 F), blood pressure is 90/60
mm Hg, pulse is 120/min and regular, and respirations are 24/min.
Examination shows marked jugular venous distention. The lungs are clear
to auscultation. Cardiac examination shows a nondisplaced and discrete
point of maximal impulse and normal S1 and S2; there is an S4 and a
right parasternal heave. Abdominal examination shows no abnormalities.
There is no edema of the lower extremities. Laboratory studies show:
Hematocrit 40%
Leukocyte count 14,000/mm3
Platelet count 350,000/mm3
Arterial blood gas analysis on 5 L/min of oxygen:
pH 7.5
PCO2 16 mm Hg
PO2 64 mm Hg
9. A previously healthy 67-year-old woman is brought to
the emergency department by paramedics 40 minutes after the sudden
onset of shortness of breath while shopping. She is unable to provide
additional medical history. She is in severe respiratory distress. Her
temperature is 37 C (98.6 F), blood pressure is 90/60 mm Hg, pulse is
120/min and regular, and respirations are 24/min. Examination shows marked
jugular venous distention. Diffuse crackles are heard throughout all
lung fields. Cardiac examination shows an enlarged point of maximal
impulse and normal S1 and S2; there is an S3. Abdominal examination shows no
abnormalities. There is no edema of the lowerextremities. Laboratory studies sho
w:
Hematocrit 38%
Leukocyte count 12,000/mm3
Platelet count 350,000/mm3
Arterial blood gas analysis on 5 L/min of oxygen:
pH 7.5
PCO2 16 mm Hg
PO2 64 mm Hg

10. A mentally competent 76-year-old man is in the


terminal stage of multiple myeloma. He is unable to move and requires
24-hour nursing care. Increasing doses of narcotics are needed to
control severe pain. As a result, when he is pain-free, respiratory

function is impaired and consciousness is clouded. The patient says he cannot


live with this degree of pain and asks to be given a lethal injection
of pain medication. Which of the following is the most
appropriate step regarding the pain medication?
A) Reduce the dosage so as not to impair respiration
B) Administer the dosage necessary to control pain despite respiratory impairmen
t
C) Administer the dosage necessary to control pain and add a centrally acting st
imulant
D) Appeal to the family to convince the patient to tolerate a bit more pain

11. A 2-day-old newborn is brought to the physician because of a


generalized rash for 6 hours. The newborn is active, alert, and feeding
well. His temperature is 36.9 C (98.4 F). Examination shows a rash
consisting of numerous white and pale yellow papules with a large base of
macular erythema over the trunk and extremities. Wright's stain of
scrapings from the lesions shows eosinophils. Which of the following is
the most appropriate next step in management?
A) Reassurance
B)Topical corticosteroid therapy
C) Intravenous acyclovir therapy
D) Intravenous ampicillin and gentamicin therapy
E) Intravenous nafcillin therapy

12. A 4-year-old boy is brought for a well-child


examination. He uses two-word phrases, can say his first name but not
his last name, and cannot identify colors. He is just beginning toilet
training. His 7-year-old sister has a learning disability and
attends special education classes. Genital development is Tanner stage 1;
testes are large. Which of the following is the most appropriate next
step in diagnosis?
A) Reexamination in 6 months
B) Thyroid function tests
C) DNA testing
D) Measurement of bone age
E) CT scan of the head

13. A 72-year-old man is brought to the physician


because of a 2-day history of nausea and vomiting. The vomitus has been
clear, and no blood has been noted. He has had a decreased appetite
for the past week. There is no associated pain or altered bowel function.
He reports that he is not seeing things correctly. He takes warfarin
and digoxin for atrial fibrillation, hydrochlorothiazide for
hypertension, and potassium supplements that he discontinued 3 weeks ago when he
ran out of tablets. His temperature is 37 C (98.6 F), blood
pressure is 144/88 mm Hg, and pulse is 52/min and irregular. Bowel sounds are
normal. The abdomen is soft and nontender without rebound or guarding. No
organomegaly or masses are palpated. Which of the following is the
most likely diagnosis?
A) Acute pancreatitis
B) Brain tumor
C) Diabetic gastroparesis
D) Diabetic ketoacidosis
E) Drug toxicity
F) Food poisoning
G) Gastric bezoar
H) Gastric carcinoma
I) Pyloric channel ulcer
J) Small-bowel obstruction
K) Uremia

14. A 27-year-old primigravid woman at 12 weeks' gestation comes to


the emergency department 2 hours after the sudden onset of bright red
vaginal bleeding. She has not had abdominal cramping.Pelvic
examination shows a small amount of brownish blood in the posterior fornix of
the vagina. The cervix is closed. The uterus is palpable 3 cm above the
pelvic brim. Fetal heart tones are easily audible at 167/min by
Doppler. Which of the following is the most likely diagnosis?
A) Abruptio placentae
B) Ectopic pregnancy
C) Incomplete abortion
D) Placenta previa
E) Threatened abortion
F) Normal pregnancy

15. A 72-year-old man is brought to the physician


by his son because of a 4-day history of increasing confusion and memory
problems. The son says that his father's ability to function
independently has been generally declining over the past few years, and he
has become much more impaired over the past week. The patient has had at
least three to four previous episodes of a sudden decline of cognitive
functioning over the past 3 years without full recovery. He has a history
of hypertension. His blood pressure is 160/95 mm Hg without orthostatic
changes. Neurologic examination shows no focal findings.
Mini-Mental State Examination score is 21/30. Which of the following is the most
likely underlying pathophysiologic process?
A) Central nervous system demyelination
B) Central nervous system infection
C) Diffuse axonal injury
D) Diffuse cortical atrophy
E) Left temporal lobe infarction
F) Multiple, small, central nervous system
infarctions
G) Subdural hematoma

16. A 72-year-old woman comes for a routine health maintenance


examination. She has a 3-year history of occasional
loss of small amounts of urine when she coughs or sneezes. She has had no
pain or burning with urination. She has hypertension treated with
daily hydrochlorothiazide. She underwent an appendectomy at
the age of 10 years. She has three children and had uncomplicated pregnancies.
Examination shows no abnormalities except for a moderate cystocele. Which
of the following is the most likely cause of this patient's urinary symptoms?
A) Chronic infectious trigonitis
B) Large intravesical calculus
C) Obstetric trauma
D) Polycystic kidney disease
E) Spastic neurogenic bladder

17. A 47-year-old woman is brought to the emergency department by


her husband because of hallucinations and agitation for 6 hours. She has
a 10-year history of alcoholism. Her last alcoholic drink was 48 hours
ago. She is agitated and inattentive. Her temperature is 38.3 C (101
F), blood pressure is 190/120 mm Hg, and pulse is 110/min. She is
oriented to person but not to place or time. During the examination, she
shrieks, "Make the lizards go away." Which of the following is the most
appropriate initial step in management?
A) Electroencephalography
B) CT scan of the head
C) Intravenous ampicillin therapy
D) Intravenous haloperidol therapy
E) Intravenous lorazepam therapy

18. A 27-year-old woman comes to the physician


because of a 3-week history of fever, night sweats, rash on both legs,
nonproductive cough, and pain and swelling in her wrists and knees. She
has not had weight loss. Her temperature is 37.7 C (99.8 F), blood
pressure is 110/70 mmHg, pulse is 96/min, and respirations are 14/min. The
lungs are clear to auscultation. Cardiac examination shows no
abnormalities. There is swelling and warmth over the wrists and knees
bilaterally and tender red nodules on the anterior surface of both lower
extremities. An x-ray film of the chest shows bilateral hilar fullness.
Which of the following is the most likely diagnosis?
A) Carcinoma of the lung
B) Histoplasmosis
C) Hodgkin's disease
D) Sarcoidosis
E) Tuberculosis

19. A 52-year-old woman comes to the physician because of difficulty


climbing stairs for 4 months. She has also noted that her thighs hurt
when she presses on them. She has had increasing difficulty combing
her hair because she tires easily. On examination, she pushes herself
out of the chair with her arms. There is weakness of the proximal
muscles of the extremities. Which of the following is the most likely
diagnosis?
A) Cauda equina syndrome
B) Cerebellar degeneration
C) Cervical spinal cord compression
D) Diabetic polyneuropathy
E) Femoral artery insufficiency
F) Guillain-Barr syndrome
G) Lumbar spinal stenosis
H) Multiple sclerosis
I) Normal-pressure hydrocephalus
J) Polymyositis
K) Sensory neuropathy
L) Tabes dorsalis

20. A 72-year-old man comes to the physician


because of generalized weakness and night sweats for 6 months. During this
period he has had a 5-kg (11-lb) weight loss. He has had polycythemia
vera for 12 years treated with hydroxyurea and multiple phlebotomies.
Examination shows cachexia. The liver is enlarged and nontender with a
span of 13 cm; the spleen is enlarged. Hematocrit is 27%, leukocyte
count is 3200/mm3, and platelet count is 150,000/mm3. A blood smear is
shown. Which of the following is the most likely diagnosis?
A) Acute myelogenous leukemia
B) Cirrhosis of the liver
C) Hodgkin's disease
D) Miliary tuberculosis
E) Myelofibrosis

21. A 30-year-old man has had increasingly severe


low back pain since lifting a heavy object at work 3 days ago. The
pain does not radiate and is not associated with bowel or bladder problems.
He has a history of occasional stiffness and mild pain in his lower
back. There is bilateral paravertebral muscle tenderness in the lumbar region an
d limited
flexion of the lumbosacral spine. Examination of the lower extremities
shows normal muscle strength and sensation;straight-leg raising is
negative bilaterally. Deep tendon reflexes at the knees and ankles are
normal. Which of the following is the most appropriate next step in management?
A) Exercises to strengthen abdominal muscles
B) Exercises to strengthen paravertebral muscles
C) Bed rest for 5 to 7 days
D) Use of a muscle relaxant
E) Use of a nonsteroidal anti-inflammatory drug

22. A 42-year-old woman, gravida 2, para 2, has had increasing


fatigue, dyspnea, orthopnea, and paroxysmal nocturnal
dyspnea over the past
2 days. She has had several episodes of hemoptysis;
she had one
episode of pulmonary edema during pregnancy 2 years
ago. A loud S1, a
snapping sound in diastole, and a rumbling diastolic
murmur are heard at the
apex. Which of the following is the most likely cause
of her
condition?
A) Atrial myxoma
B) Bicuspid aortic valve
C) Postpartum cardiomyopathy
D) Rheumatic heart disease
E) Viral myocarditis
23. A 37-year-old woman has had unilateral breast
pain, fever, and chills for 24 hours. She breast-feeds her 1-month-old
newborn. Her temperature is 38 C (100.4 F). Examination shows
swelling, erythema, and localized tenderness of the left breast. Which of the
following is the most appropriate next step in management?
A) Application of ice packs to the affected breast

B) Use of a breast pump


C) Immediate discontinuation of breast-feeding
D) Bromocriptine therapy
E) Penicillinase-resistant antibiotic therapy
24. A 19-year-old woman, gravida 2, para 1, at 39 weeks' gestation
is admitted in labor. Contractions occur every 2 to 3 minutes. The
cervix is 4 cm dilated and 80% effaced. She requests an epidural for pain
control. Ten minutes after the epidural is administered, she becomes
nauseated and diaphoretic and vomits. Her blood pressure is 60/palpable
mm Hg. A fetal heart tracing shows sustained fetal decelerations. The
cervix is now 8 cm dilated. The most appropriate next step in
management is administration of which of the following?
A) Ephedrine
B) Magnesium sulfate
C) Nifedipine
D) Oxytocin
E) Terbutaline

25. An asymptomatic 47-year-old man comes for a


routine follow-up examination 3 weeks after sustaining an inferior wall
myocardial infarction. His recovery was complicated by transient
premature ventricular contractions during the first 2 days of
hospitalization. An exercise stress test prior to discharge showed no pain at 70
%
of his predicted maximum exercise capacity. Medications include daily
aspirin and pravastatin. His blood pressure is 136/80 mm Hg,
pulse is 80/min and regular, and respirations are 16/min. Cardiopulmonary
examination shows no abnormalities. Which of the following is the most
appropriate additional pharmacotherapy for this patient?
A) Clonidine
B) Metoprolol
C) Quinidine
D) Sulfinpyrazone
E) Verapamil

26. A 35-year-old woman is brought to the emergency


department by her family because of shortness of breath, tightness
in her chest, and palpitations for 2 hours. Over the past 11 months,
she has had five similar episodes; during the last episode 3 weeks ago,
she was treated with an intravenous medication that caused conversion to

sinus rhythm. Her blood pressure is 95/60 mm Hg, and pulse is 165/min
and regular. The lungs are clear to auscultation. Which of the
following is the most likely underlying dysrhythmia?
A) Accelerated idioventricular rhythm
B) Accelerated junctional rhythm
C) Atrial fibrillation
D) Multifocal atrial tachycardia
E) Normal sinus rhythm
F) Paroxysmal supraventricular tachycardia
G) Premature supraventricular beats
H) Premature ventricular beats
I) Sick sinus syndrome
J) Sinus bradycardia
K) Sinus tachycardia
L) Ventricular fibrillation
M) Ventricular tachycardia

27. A 4-year-old boy is brought to the emergency


department because of a 1-day history of fever and increasing difficulty
breathing. Over the past year, he has had recurrent bacterial infections
including cervical lymphadenitis, septic arthritis, and pneumonia. Histemperatur
e is 38.9
C (102 F), blood pressure is 80/60 mm Hg, pulse is115/min, and
respirations are 38/min and labored. Breath sounds are decreased over the
left anterior chest. Scattered crackles are heard on auscultation. A
thoracentesis shows purulent fluid. Laboratory studies show:
Hematocrit 36%
Leukocyte count 18,000/mm3
Segmented neutrophils 85%
Lymphocytes 15%
Platelet count 200,000/mm3
Pleural fluid
Leukocyte count 75,000/mm3
Segmented neutrophils 98%
Lymphocytes 2%
Nitroblue tetrazolium test is abnormal. A Gram's stain of the pleural
fluid shows numerous gram-positive cocci in the segmented neutrophils.
An x-ray film of the chest shows left lower lobe pneumonia with pleural
effusion. The most likely cause of this patient's symptoms is a defect
of which of the following?

A) Chemotaxis
B) Immotile cilia
C) Opsonization
D) Phagocytic oxidative metabolism
E) Phagocytosis
F) T-lymphocyte function

28. A 59-year-old woman comes to the emergency


department 45 minutes after the onset of chest discomfort that is not
relieved by three nitroglycerin tablets. Over the past 3 months, she
has had similar episodes characterized by nonradiating pain and a feeling of
heaviness; the episodes were exacerbated by exertion or heavy meals
and were slowly relieved by rest. Sublingual nitroglycerin has
provided rapid relief of symptoms in the past. She has hypercholesterolemia,
type 2 diabetes mellitus, and peptic ulcer disease. She smoked two
packs of cigarettes daily for 25 years but quit 5 years ago. She appears
anxious and diaphoretic and is nauseated. Examination shows no other
abnormalities except for an S4. Which of the following is the most likely
diagnosis?
A) Acute aortic dissection
B) Angina pectoris
C) Esophageal spasm
D) Myocardial infarction
E) Pulmonary embolism

29. A 27-year-old woman comes to the physician


because of increasingly severe daily headaches over the past 3
months. The headaches are diffuse and often occur at the base of the skull. For
the past month, she has had brief episodes of visual darkening when
standing. She has been amenorrheic for the past year and has had an18-kg (40-lb)
weight
gain during this period. She now weighs 118 kg (260lb) and is 152 cm
(60 in) tall. Funduscopic examination shows
papilledema, several flame hemorrhages, and an enlarged blind spot bilaterally.
Visual acuity is 20/20 bilaterally. A CT scan of the head shows no
abnormalities. Which of the following is the most likely underlying cause
of these findings?
A) Cerebral venous occlusion
B) Communicating hydrocephalus
C) Cytotoxic edema
D) Idiopathic intracranial hypertension

E) Impaired absorption of cerebrospinal fluid


F) Infratentorial mass lesion
G) Interstitial edema
H) Overproduction of cerebrospinal fluid
I) Vasogenic edema

30. A 75-year-old man with a 3-year history of progressive cognitive


impairment due to dementia, Alzheimer's type, has had nocturnal
disorientation for 2 weeks. He lives at home with his wife. He is otherwise
healthy and takes no medications. Physical examination shows normal
findings. He is disoriented to time and place, has
poor short-term memory, is unable to do simple arithmetic, and has a poor unders
tanding of
general information. Which of the following is the most appropriate
initial step in management?
A) Increase in home nighttime lighting
B) Prescription for chloral hydrate
C) Prescription for diazepam
D) Prescription for haloperidol
E) Use of nighttime mechanical restraints

31. A 62-year-old woman comes to the physician because of severe


pain and swelling of her right knee for 1 day. She has no history of
joint disease or trauma to the knee. She has hypertension treated with
hydrochlorothiazide and type 2 diabetes mellitus treated with glyburide.
She is sexually active only with her husband, and they have sexual
intercourse one to two times each week. Her temperature is 37 C (98.6 F),
blood pressure is 140/84 mm Hg, and pulse is 80/min. Examination of
the right knee shows edema, erythema, and exquisite tenderness to light
touch; there is an effusion. The remainder of the examination shows no
abnormalities. Which of the following is the most likely mechanism of
these findings?
A) Immune complex deposition
B) Inflammatory reaction to antisynovial antibodies
C) Inflammatory reaction to monosodium urate

crystals
D) Neisseria gonorrhoeae infection
E) Streptococcus pneumoniae infection

32. Two days after a cholecystectomy, a 42-year-old


woman has shortness of breath. Her temperature is 37.5 C (99.5F), blood pressure
is 110/70 mm Hg, pulse is 103/min, and respirations are 24/min. There is
abdominal tenderness. Examination shows no other abnormalities. An x-ray film of
the chest shows minimal linear markings in the right lower
lobe. Arterial blood gas analysis on 45% oxygen shows:
pH 7.41
PCO2 40 mm Hg
PO2 52 mm Hg
Which of the following is the most likely explanation
for her hypoxia?
A) Acute respiratory distress syndrome
B) Atelectasis
C) Congestive heart failure
D) Fat embolism syndrome
E) Pneumonia
F) Pneumothorax

33. A 37-year-old woman comes to the emergency department because of


a 3-day history of increasingly severe abdominal pain, nausea, and
vomiting. Twelve years ago, she had a hysterectomy because of severe
dysfunctional uterine bleeding. Her temperature is 37C (98.6 F), blood
pressure is 106/70 mm Hg, pulse is 110/min, and respirations are 12/min.
Examination shows a distended, tympanic abdomen with diffuse tenderness
but no guarding; bowel sounds are hypoactive. Her leukocyte count is
10,000/mm3, and hematocrit is 44%. An x-ray film of the abdomen is
shown. Which of the following is the most appropriate initial step in

management?
A) CT scan of the abdomen
B) Intravenous neostigmine therapy
C) Esophagogastroduodenoscopy
D) Nasogastric intubation
E) Laparotomy

34. A healthy 27-year-old woman comes for a routine health


maintenance examination. Her blood pressure is 185/90mm Hg. Examination shows
no other abnormalities except for hypertensive retinopathy. Serum
studies show a sodium level of 140 mEq/L, potassium level of 4 mEq/L, and
creatinine level of 1.1 mg/dL. A complete blood count, serum
catecholamine levels, and urinalysis are within normal limits. At two subsequent
visits, her blood pressure is 190/100 mm Hg and 182/96mm Hg,
respectively. Which of the following is the most appropriate next step in diagno
sis?
A) 24-Hour urine collection for measurement of creatinine clearance
B) 24-Hour urine collection for measurement of 17-hydroxycorticosteroid and tot
al 17-ketosteroid levels
C) Measurement of serum aldosterone level
D) Magnetic resonance angiography of renal vessels
E) CT scan of the abdomen

35. A 76-year-old man has had fatigue and loss of interest in daily activities o
ver the past 4 months. He sleeps poorly and has had a
4.5-kg (10-lb) weight loss during this period. He states that he has
probably lived long enough. His blood pressure is 110/78 mm Hg, and pulse
is 68/min. Examination shows a slow return of deep tendon reflexes.
Measurement of which of the following serum levels is the most
appropriate next step in management?
A) Calcium
B) Creatinine
C) Glucose
D) Testosterone
E) Thyroid-stimulating hormone

36. A 29-year-old woman is brought to the physician by her father.


She has been working continuously without sleep for 3 days on a project
which she claims "Allah and Jesus have told me to do" that will "unify
all knowledge in physics and medicine and philosophy. "She hears the
voice of her dead brother telling her that she will be "the next Virgin
Mary." Her father states that she was treated for depression for 1 year
at the age of 19 years. Which of the following is the most likely
diagnosis?
A) Bipolar disorder
B) Borderline personality disorder
C) Major depressive disorder
D) Schizophrenia
E) Substance-induced mood disorder

37. A 67-year-old man is hospitalized for treatment of renal


insufficiency. Three days after admission, his pulse is 40/min. An ECG shows
tall, tented T waves. Serum studies show a sodium level of 134 mEq/L,
potassium level of 6.9 mEq/L, and glucose level of 85mg/dL. The most
appropriate next step in management is intravenous administration of
which of the following?
A) Calcium, furosemide, and 3% saline
B) Calcium, insulin, and digitalis
C) Calcium, insulin, and glucose
D) Glucose, furosemide, and phosphate
E) Glucose, glucagon, and bicarbonate
38. Six months after the delivery of her fourth child, a 37-year-old
woman undergoes laparoscopic tubal ligation. Menses occur at regular
28-day intervals. During the operation, she is found to have a small
dark lesion in the cul-de-sac and filmy adhesions surrounding the
ovaries. A biopsy specimen of a cul-de-sac lesion confirms the diagnosis of
endometriosis. Which of the following is the most appropriate next step
in management?
A) Danazol therapy
B) Gonadotropin-releasing hormone agonist therapy
C) Oral contraceptive therapy
D) Total abdominal hysterectomy and bilateral
salpingo-oophorectomy
E) No further treatment indicated

A)
B)
C)
D)
E)
F)
G)
H)

Administration of parenteral antibiotics


Admission to the hospital for medical management
Admission to the hospital for operative management
Colon contrast studies
Discharge for follow-up by personal physician
Endoscopy
MRI of the abdomen
Observation in the emergency department

39. A previously healthy 6-month-old boy is brought to the emergency


department because of a 12-hour history of intermittent episodes of
inconsolable crying associated with drawing up of the legs. Over the past
6 hours, he has had intermittent diarrhea that is reddish and mucoid,
and for the past 3 hours he has been somnolent. On examination, he is
sleepy but arousable. His temperature is 38.1 C (100.6 F), blood
pressure is 90/55 mm Hg, pulse is 140/min, and respirations are 38/min. He
cries when his abdomen is palpated; a mass is felt in the right lower
quadrant. His leukocyte count is 12,400/mm3 (50% segmented neutrophils,
8% bands, 1% eosinophils, 40% lymphocytes, and 1%
monocytes). Serum electrolyte levels are within normal limits. An x-ray
film of the abdomen shows no free air.

40. A previously healthy 14-year-old girl is brought to the


emergency department because of abdominal pain for 12 hours. She has a 1-week
history of brownish vaginal discharge. Menarche was at the age of 12
years, and her periods have occurred at regular 28-day intervals over
the past year. Her last menstrual period was 7 weeks ago. Her
temperature is 37 C (98.6 F), blood pressure is 85/55
mm Hg, pulse is 145/min, and respirations are 24/min. Abdominal examination
shows generalized tenderness, and there is guarding with rebound in the
right lower quadrant. Her hematocrit is 24%, and leukocyte count
is 9400/mm3 (60% segmented neutrophils, 3% bands, 1% eosinophils, 35%
lymphocytes, and 1% monocytes). Serum electrolyte levels are within normal limit
s.

41. A 49-year-old woman is admitted to the hospital because of renal


failure. She has had episodes of flank pain over the past 20 years.
She has also had nocturia 2 to 3 times nightly for 10 years. Her blood
pressure is 160/100 mm Hg. Examination shows pale mucous membranes. A
mass is palpated in the right flank. Which of the following is the
most likely diagnosis?
A) Horseshoe kidney
B) Nephrolithiasis
C) Papillary necrosis
D) Polycystic kidney disease
E) Renal cell carcinoma
42. On routine annual screening, an asymptomatic 27-year-old man has
a positive PPD skin test. One year ago, a PPD skin test was negative.
He works as a nurse. Three years ago, he was diagnosed with hepatitis
A after a trip to South America. Examination shows no abnormalities.
His serum aspartate aminotransferase (AST, GOT) activity is 10 U/L, and
serum alanine aminotransferase (ALT, GPT) activity is 14 U/L. An x-ray
film of the chest shows no abnormalities. Which of the following is
the most appropriate chemoprophylaxis?
A) Isoniazid and folic acid supplementation
B) Isoniazid and rifampin
C) Isoniazid and vitamin B1 (thiamine) supplementation
D) Isoniazid and vitamin B6 supplementation

E) No prophylaxis indicated
43. A 4-month-old boy is brought to the physician because
of a 2-day history of fever and progressive redness around his
right eye. He has had persistent diarrhea and oral candidiasis since birth
and was treated for pneumococcal pneumonia at the age of 2 months. He
appears ill. His temperature is 39 C (102.2 F), pulse is 130/min, and
respirations are 25/min. Examination shows violaceous preseptal(periorbital) cel
lulitis
and oral candidiasis. Laboratory studies show:
Hemoglobin 10 g/dL
Leukocyte count 3000/mm3
Segmented neutrophils 85%
Lymphocytes 15%
Platelet count 350,000/mm3
Serum
IgA <5 mg/dL
IgG 300 mg/dL
IgM <5 mg/dL
Which of the following is the most likely diagnosis?
A) AIDS
B) Chronic granulomatous disease
C) Severe combined immunodeficiency
D) Thymic-parathyroid dysplasia (DiGeorge syndrome)
E) X-linked agammaglobulinemia
44. One week after undergoing an uncomplicated liver transplant for
biliary atresia, a 3-year-old boy appears jaundiced. Examination shows
scleral icterus. His serum aspartate aminotransferase(AST, GOT)
activity has increased to 1300 U/L, and serum alanine aminotransferase (ALT,
GPT) activity has increased to 2500 U/L. His serum bilirubin level is
3.5 mg/dL, and serum alkaline phosphatase activity is 100 U/L. Which
of the following is the most likely artery responsible
for this patient's gastrointestinal symptoms?
A) Hepatic
B) Ileocolic
C) Inferior mesenteric
D) Left gastric
E) Left gastroepiploic
F) Middle colic
G) Posterior penetrating
H) Right colic
I) Right gastroepiploic
J) Splenic
K) Superior hemorrhoidal

L) Superior mesenteric

45. A 2-year-old girl has had fever and bloody diarrhea for 10 days.
A stool culture obtained 7 days ago grew Salmonella species sensitive
to amoxicillin. A blood culture was negative. Despite beginning oral
amoxicillin therapy 4 days ago, her diarrhea has persisted. Current
examination shows no other abnormalities except for a temperature of 38.6 C
(101.5 F). Which of the following is the most likely explanation for
the failure of amoxicillin to improve her symptoms?
A) Amoxicillin does not alter the course of Salmonella enteritidis
B) Amoxicillin has caused pseudomembranous colitis
C) Amoxicillin is absorbed at the level of the jejunum, leaving no drug to be de
livered to the colon
D) Oral amoxicillin is not absorbed into the systemic circulation in the presenc
e of diarrhea
E) Salmonella has expressed an inducible -lactamase that inactivates amoxicillin

46. A 23-year-old man comes to the physician because of a


1-month history of intermittent right-sided abdominal pain, nausea,
and vomiting. He has sickle cell disease and has been treated several
times for painful crises. Examination of the abdomen shows tenderness
to palpation of the right upper quadrant on inspiration. Laboratory
studies show:
Hematocrit 25%
Leukocyte count 11,000/mm3
Serum
Bilirubin
Total 3.2 mg/dL
Direct 0.3 mg/dL
Alkaline phosphatase 56 U/L
Ultrasonography of the gallbladder shows a filling defect. Which of the followin
g is the most likely cause of this patient's hyperbilirubinemia?
A) Aggregation of cholesterol in the gallbladder
B) Inhibition of glucuronosyltransferase
C) Lysis of erythrocytes
D) Malnutrition-induced cirrhosis
E) Neoplastic growth in the gallbladder

BLOCK 1
1. 1. e
2. a
3. B
4. E
5. a? b?
6. b
7. b
8. C
9. D
10. A
11. J
12. H
13. d
14. a
15. d
16. b
17.
18. d
19. A
20. c
21. A
22. B
23. a
24. b
25. B/D
26. A
27. e
28. C
29. D
30. G
31. D
32. B
33. E
34. A
35. c
36. a
37. f
38.A
39. G
40. a
41. E
42. B
43. E
44. D
45. B
46. F
BLOCK 2
1 .D
2.B
3.A
4. G??
5. C
6. C
7.B
8.D
9.A

10.C??
11.C
12.D
13. C
14.C
15.A
16.C
17. A /?? E
18.B
19.B
20.E
21.E
22.D
23.D
24. E
25.C
26.F
27.B
28.E
29.B
30.D
31.E
32.B
33.A
34.A
35.E
36.E
37.E
38.F
39.D
40.B ??
41.E
42.D
43.E
44. E
45. C( OSTEOMYLITIS)
46.D
BLOCK 3
1.d
2.c
3.c
4.b
5.a
6.a
7.b
8.d
9.c
10.b
11.e
12.b
13.d
14.a
16.e
17.c
18.g
19.c
20.b

21.e
22.b
23.c
24.b
25.h
26.e
27.c
28.d?
29.g
30.a
31.d.
32.d
33.c
34.c
35.b
36.c.
37.b
38.d
39.a
40.d
41.e.
42.f
43.d
44.b
45.c
46.d
BLOCK 4
1.a
2.a
3.b?
4. e/a
5.e/b
6.f
7.c
8.c/g
9.g
10.a/c.. very controvertial plz explain
11.a
12.c
13.e
14.e
15.f
16.c
17.e/d
18.d
19.j
20.a /? E
21.e/d
22.d
23.a
24.a?
25.b
26.f
27.d
28.d
29.d
30.a
31.b

32.b
33.d nosogastric suction
34.d
35.e
36.a
37.b
38e?
39.d
40.d
41.d
42.d
43.c
44.a
45.b??
46.c

S-ar putea să vă placă și